Вы находитесь на странице: 1из 78

Exam Title : GS Simulator Test 5

Email

Exam URL : http://compete.forumias.com

1.

The Jataka stories mainly consist of


a) Preachings of Lord Buddha through animal tales
b) Stories of previous births of Buddha
c) Canons of Buddhism
d) Tales written by later Buddhists
Correct Answer: B
Your Answer: Unanswered
Explanation:
Explanation 1 - The Jataka tales are a voluminous body of literature native to India concerning the
previous births of Gautama Buddha. These are the stories that tell about the previous lives of the
Buddha, in both human and animal form. The stories come from multiple sources and have a multitude
of authors. Like other Buddhist literature, the many stories can be divided into "Theravada" and
"Mahayana" canons.
In Theravada Buddhism, the Jatakas are a textual division of the Pali Canon, included in the Khuddaka
Nikaya of the Sutta Pitaka .
Each tale begins by noting the occasion that prompted its telling and ends with the Buddha identifying
the lives of the people in the introductory story with those of people from the past. There is humour in
these stories and considerable variety. Many Jatakas have parallels in the Mahabharata (Great Epic of
the Bharata Dynasty), the Panchatantra (animal fables), the Puranas ( collections of legends), and
elsewhere in non-Buddhist Indian literature. The Jataka stories have also been illustrated frequently in
sculpture and painting throughout the Buddhist world.

2.

The term homeostasis refers to the


a)
b)
c)
d)

Capacity of ecosystem to make food available for all habitats


Capacity of ecosystem to move to secondary succession
Capacity of ecosystem of self-regulation
None of the above

Correct Answer: C
Your Answer: Unanswered
Explanation:
(c) Ecosystems are capable of maintaining their state of equilibrium. They can regulate their own species
structure and functional processes. This capacity of ecosystem of self-regulation is known as
homeostasis. In ecology the term applies to the tendency for a biological systems to resist changes.

ForumIAS
Web: http://www.forumias.com/
Email: ravi@forumias.com

Page 1

Score: 0.00 / 200

Exam Title : GS Simulator Test 5


Email

Exam URL : http://compete.forumias.com

3.

Payments Banks have been in news recently. In this context, consider the following statements
1. India Post has been given license to operate small finance banks by RBI.
2. Payment banks can accept maximum deposit of Rs . 1 lakhs per individual.
3. Small finance banks will be able to give credit to small business units.
Select the correct statement(s) using the codes given below:
a) 1 and 2 only
b) 2 and 3 only
c) 1 and 3 only
d) 1, 2 and 3
Correct Answer: B
Your Answer: Unanswered
Explanation:

ForumIAS
Web: http://www.forumias.com/
Email: ravi@forumias.com

Page 2

Score: 0.00 / 200

Exam Title : GS Simulator Test 5


Email

Exam URL : http://compete.forumias.com

Explanation 3 : Primary objective of setting up of payments banks is to further financial inclusion by


providing
( i ) small savings accounts and
(ii) payments / remittance services to migrant labour workforce, low income households, small
businesses, other unorganised sector entities and other users, by enabling high volume-low value
transactions in deposits and payments / remittance services in a secured technology-driven
environment. Payment banks can not give loans. Payment banks can accept deposits restricted to Rs . 1
lakh per customer, and are allowed to pay customers interest on the money that is being deposited.
They can be used for either current accounts or savings accounts.
The payments bank are registered as a public limited company under the Companies Act, 2013, and
licensed under Section 22 of the Banking Regulation Act, 1949, with specific licensing conditions
restricting its activities mainly to acceptance of demand deposits and provision of payments and
remittance services. It will be governed by the provisions of the Banking Regulation Act, 1949; Reserve
Bank of India Act, 1934; Foreign Exchange Management Act, 1999; Payment and Settlement Systems
Act, 2007; Deposit Insurance and Credit Guarantee Corporation Act, 1961; other relevant Statutes and
Directives, Prudential Regulations and other Guidelines/Instructions issued by RBI and other regulators
from time to time.
Small finance bank are set to undertake basic banking activities of acceptance of deposits and lending to
unserved and underserved sections including small business units, small and marginal farmers, micro
and small industries and unorganised sector entities.The minimum paid-up equity capital for small
finance banks shall be Rs . 100 crore.
Key objectives of setting up of small finance banks will be to further financial inclusion by
(a) provision of savings vehicles, and
(ii) supply of credit to small business units; small and marginal farmers; micro and small industries; and
other unorganised sector entities, through high technology-low cost operations.
4.

With reference to Indias culture and tradition, what is Thang Ta?


a) It is a Marshal Art dance-form of Manipur.
b) It is an ancient style bronze and brass-work found in coromandal area.
c) It is a form of music developed by producing musical nodes on brass plates.
d) It is an ancient Bhakti Cult of Shaivism still prevalent in some parts of South India.
Correct Answer: A
Your Answer: Unanswered
Explanation:

ForumIAS
Web: http://www.forumias.com/
Email: ravi@forumias.com

Page 3

Score: 0.00 / 200

Exam Title : GS Simulator Test 5


Email

Exam URL : http://compete.forumias.com

(a) Thang Ta dance is a weapon based martial dance form of its state Manipur. This dance was created
by Meitei people in Manipur region. Also this dance name is derived from two words which are
commonly known as Thang means sword and ta means sword. As from the name this dance form name
means it is a sword and spear dance form and dancers make use of these primary weapons in their
dance form. The sword used in it is in non-missile form while when it is close and is thrown afar even.
Some other weapons are even used in this dance form such as axe and shield.
5.

Consider the following statements about FDI norms, recently in news in India
1. FDI in single brand retail are subject to 30% local sourcing norms.
2. 100% FDI in online eCommerce is allowed under the automatic route
Select the correct statements using the codes give below a)
b)
c)
d)

1 only
2 only
Both 1 & 2
None of the Above

Correct Answer: B
Your Answer: Unanswered
Explanation:
Current FDI norms require 30% local sourcing in single brand retail FDI. Tim Cook, Apple CEO has
recently argued for relaxing this, but the government has refused any relaxation. In the online
marketplace model , 100% FDI is now allowed. There is also a new provision where a single vendor on
an ecommerce platform cannot have more than 25% of sales.
6.

Consider the following.


1. Eri
2. Tasar
3. Mulberry
4. Muga
Which of the above is/are the varieties of natural silk?
a) 3 only
b) 3 and 4 only
c) 1, 3 and 4 only
d) 1, 2, 3 and 4
Correct Answer: D
Your Answer: Unanswered
Explanation:

ForumIAS
Web: http://www.forumias.com/
Email: ravi@forumias.com

Page 4

Score: 0.00 / 200

Exam Title : GS Simulator Test 5


Email

Exam URL : http://compete.forumias.com

(d) There are four types of natural silk which are commercially known and produced in the world. Among
them mulberry silk is the most important and contributes as much as 90 per cent of world production,
therefore, the term "silk" in general refers to the silk of the mulberry silkworm. Three other commercially
important types fall into the category of non-mulberry silks namely: Eri silk; Tasar silk; and Muga silk.
There are also other types of non-mulberry silk, which are mostly wild and exploited in Africa and Asia,
are Anaphe silk, Fagara silk, Coan silk, Mussel silk and Spider silk.
7.

Which among the following is/are the methods soil erosion by wind?
1. Siltation
2. Suspension
3. Surface Creep
Select the correct answer using the codes given below:
a) a ) 1 and 2 only
b) 1, 2 and 3
c) 2 and 3 only
d) 1 and 3 only
Correct Answer: B
Your Answer: Unanswered
Explanation:

ForumIAS
Web: http://www.forumias.com/
Email: ravi@forumias.com

Page 5

Score: 0.00 / 200

Exam Title : GS Simulator Test 5


Email

Exam URL : http://compete.forumias.com

Explanation 7 : Types of soil erosion by wind :


SUSPENSION: Fine particles less than 0.1 mm in size are moved parallel to the surface and upward into the atmosphere by strong winds. The most spectacular of erosive processes, these particles can be carried high into the
atmosphere, returning to earth only when the wind subsides or they are carried downward with precipitation. Suspended particles can travel hundreds of miles.

2. SALTATION: Movement of particles by a series of short bounces along the surface of the ground, and dislodging additional particles with each impact. The bouncing particles ranging in size from 0.1 to 0.5 mm usually remain within
30 cm of the surface. Depending on conditions, this process accounts for 50 to 90% of the total movement of soil by wind.
3. SOIL CREEP: The rolling and sliding of larger soil particles along the ground surface. The movement of these particles is aided by the bouncing impacts of the saltating particles described above. Soil creep can move particles
ranging from 0.5 to 1 mm in diameter, and accounts for 5 to 25% of total soil movement by wind.
Soil erosion is a naturally occurring process that affects all landforms. In agriculture, soil erosion refers to the wearing away of a field's topsoil by the natural physical forces of water and wind or through forces associated with farming
activities such as tillage.
Erosion, whether it is by water, wind or tillage, involves three distinct actions soil detachment, movement and deposition. Topsoil, which is high in organic matter, fertility and soil life, is relocated elsewhere "on-site" where it builds up
over time or is carried "off-site" where it fills in drainage channels. Soil erosion reduces cropland productivity and contributes to the pollution of adjacent watercourses, wetlands and lakes.
Soil erosion can be a slow process that continues relatively unnoticed or can occur at an alarming rate, causing serious loss of topsoil. Soil compaction, low organic matter, loss of soil structure, poor internal drainage, salinisation and
soil acidity problems are other serious soil degradation conditions that can accelerate the soil erosion process.

Water Erosion: The widespread occurrence of water erosion combined with the severity of on-site and off-site impacts have made water erosion the focus of soil conservation efforts in Ontario.
The rate and magnitude of soil erosion by water is controlled by the following factors:
Rainfall and Runoff
The greater the intensity and duration of a rainstorm, the higher the erosion potential. The impact of raindrops on the soil surface can break down soil aggregates and disperse the aggregate material. Lighter aggregate materials such
as very fine sand, silt, clay and organic matter are easily removed by the raindrop splash and runoff water; greater raindrop energy or runoff amounts are required to move larger sand and gravel particles.
Soil movement by rainfall (raindrop splash) is usually greatest and most noticeable during short-duration, high-intensity thunderstorms. Although the erosion caused by long-lasting and less-intense storms is not usually as spectacular
or noticeable as that produced during thunderstorms, the amount of soil loss can be significant, especially when compounded over time.
Soil Erodibility
Soil erodibility is an estimate of the ability of soils to resist erosion, based on the physical characteristics of each soil. Texture is the principal characteristic affecting erodibility, but structure, organic matter and permeability also
contribute. Generally, soils with faster infiltration rates, higher levels of organic matter and improved soil structure have a greater resistance to erosion. Sand, sandy loam and loam-textured soils tend to be less erodible than silt, very
fine sand and certain clay-textured soils.
Tillage and cropping practices that reduce soil organic matter levels, cause poor soil structure, or result in soil compaction, contribute to increases in soil erodibility. As an example, compacted subsurface soil layers can decrease
infiltration and increase runoff. The formation of a soil crust, which tends to "seal" the surface, also decreases infiltration. On some sites, a soil crust might decrease the amount of soil loss from raindrop impact and splash; however, a
corresponding increase in the amount of runoff water can contribute to more serious erosion problems.
Slope Gradient and Length
The steeper and longer the slope of a field, the higher the risk for erosion.
Cropping and Vegetation
The potential for soil erosion increases if the soil has no or very little vegetative cover of plants and/or crop residues. Plant and residue cover protects the soil from raindrop impact and splash, tends to slow down the movement of runoff
water and allows excess surface water to infiltrate.
The erosion-reducing effectiveness of plant and/or crop residues depends on the type, extent and quantity of cover. Vegetation and residue combinations that completely cover the soil and intercept all falling raindrops at and close to
the surface are the most efficient in controlling soil erosion (e.g., forests, permanent grasses). Partially incorporated residues and residual roots are also important as these provide channels that allow surface water to move into the soil.
Tillage Practices
The potential for soil erosion by water is affected by tillage operations, depending on the depth, direction and timing of plowing , the type of tillage equipment and the number of passes. Generally, the less the disturbance of vegetation
or residue cover at or near the surface, the more effective the tillage practice in reducing water erosion. Minimum till or no-till practices are effective in reducing soil erosion by water

Wind Erosion
Wind erosion occurs in susceptible areas of Ontario but represents a small percentage of land mainly sandy and organic or muck soils. Under the right conditions it can cause major losses of soil and property
Soil particles move in three ways, depending on soil particle size and wind strength suspension, saltation and surface creep.
The rate and magnitude of soil erosion by wind is controlled by the following factors:
Soil Erodibility
Very fine soil particles are carried high into the air by the wind and transported great distances (suspension). Fine-to-medium size soil particles are lifted a short distance into the air and drop back to the soil surface, damaging crops and
dislodging more soil (saltation). Larger-sized soil particles that are too large to be lifted off the ground are dislodged by the wind and roll along the soil surface (surface creep). The abrasion that results from windblown particles breaks
down stable surface aggregates and further increases the soil erodibility.
Soil Surface Roughness
Soil surfaces that are not rough offer little resistance to the wind. However, ridges left from tillage can dry out more quickly in a wind event, resulting in more loose, dry soil available to blow. Over time, soil surfaces become filled in, and
the roughness is broken down by abrasion. This results in a smoother surface susceptible to the wind. Excess tillage can contribute to soil structure breakdown and increased erosion.
Climate
The speed and duration of the wind have a direct relationship to the extent of soil erosion. Soil moisture levels are very low at the surface of excessively drained soils or during periods of drought, thus releasing the particles for transport
by wind. This effect also occurs in freeze-drying of the soil surface during winter months. Accumulation of soil on the leeward side of barriers such as fence rows, trees or buildings, or snow cover that has a brown colour during winter
are indicators of wind erosion.
Unsheltered Distance
A lack of windbreaks (trees, shrubs, crop residue, etc.) allows the wind to put soil particles into motion for greater distances, thus increasing abrasion and soil erosion. Knolls and hilltops are usually exposed and suffer the most.
Vegetative Cover
The lack of permanent vegetative cover in certain locations results in extensive wind erosion. Loose, dry, bare soil is the most susceptible; however, crops that produce low levels of residue (e.g., soybeans and many vegetable crops)
may not provide enough resistance. In severe cases, even crops that produce a lot of residue may not protect the soil.
The most effective protective vegetative cover consists of a cover crop with an adequate network of living windbreaks in combination with good tillage, residue management and crop selection.

ForumIAS
Web: http://www.forumias.com/
Email: ravi@forumias.com

Page 6

Score: 0.00 / 200

Exam Title : GS Simulator Test 5


Email

Exam URL : http://compete.forumias.com

8.

Which among the following cess is/are applicable on service tax?


1. Krishi Kalyan Cess
2. Swacchh Bharat Cess
3. Education Cess
Select the correct answer using the codes given below:
a) a ) 1 and 3 only
b) 1 and 2 only
c) 2 and 3 only
d) 1, 2 and 3
Correct Answer: B
Your Answer: Unanswered
Explanation:
Explanation 8 : Service tax rate has tripled in 22 years. Let us see a brief history of it. Service Tax was
first introduced in 1994. During his budget speech for the financial year 1994-95, the then Finance
Minister Dr Manmohan Singh emphasized the need for service tax. He said that though the services
sector accounted for 40 percent of the GDP, it was never taxed. Based on the recommendations of the
tax reforms committee, he proposed to impose service tax of 5 percent only on 3 services namely
telephone bills, non-life insurance and tax brokers.
The service tax regime started with 3 services under the tax net in 1994 and progressively increased to
119 different services in 2011-12. The latest to be included in 2011-12 were services of air conditioned
restaurants and hotel accommodation.
From 2012, there has been a paradigm shift in the levy of service tax. Instead of levying tax on certain
services, tax was imposed on all services except those listed in the negative list.
In 2006, the service tax rate was increased from 10 percent to 12 percent. In addition, a Secondary &
Higher Education Cess of 1 percent on service tax was introduced. The net service rate became 12.36
percent. It remained at 12.36 percent till 2009 when it was brought back to 10.3 percent under the
expectation of introducing GST from 2010. In 2012, the base service rate was increased again from 10
percent to 12 percent while the overall service tax rate increased to 12.36 percent. It remained at 12.36
percent till 2015 when the NDA government increased the service rate to 14 percent including the
education cess. In addition, Swachh Bharat Cess of 0.5 percent was also introduced taking the
effective service tax rate to 14.5 percent. In 2016-17, while the base rate remained unchanged, Krishi
Kalyan Cess of 0.5 percent was introduced taking the service tax rate to 15 percent.

ForumIAS
Web: http://www.forumias.com/
Email: ravi@forumias.com

Page 7

Score: 0.00 / 200

Exam Title : GS Simulator Test 5


Email

Exam URL : http://compete.forumias.com

9.

Consider the following actions taken by Parliament.


1. Removal of Comptroller and Auditor General
2. Approval of emergency proclamation
3. No Confidence motion
4. Constitutional amendment
In which of the above cases, concurrence of both Lok Sabha and Rajya Sabha is required?
a) 2 and 4 only
b) 1, 2 and 4 only
c) 1, 2 and 3 only
d) 1, 3 and 4 only
Correct Answer: B
Your Answer: Unanswered
Explanation:
(b) b) No Confidence motion is only introduced in Loksabha . RajyaSabha has no role in this.
CAG can be removed by same manner as a judge of the SC. Thus both houses are involved.
Emergency.
Proclamation of emergency must be approved by both houses of the parliament within one month of its
issue.
Constitutional amendment involves two houses.

10. Consider the following statements regarding biodiversity hotspots.


1. The concept originated to designate priority areas for in situ conservation.
2. The area should support greater than 1800 species
3. It must have lost over 70% of original habitat.
Select the correct statement(s) using the codes given below:
a)
b)
c)
d)

a ) 1 and 2 only
2 and 3 only
1 and 3 only
1, 2 and 3

Correct Answer: C
Your Answer: Unanswered
Explanation:
(c) The statement 1 was the objective of creating biodiversity hotspots while statements 2 and 3 are the
criteria for selecting biodiversity hotspots.
Statement 2 is wrong as the number should be 1500 in place of 1800

ForumIAS
Web: http://www.forumias.com/
Email: ravi@forumias.com

Page 8

Score: 0.00 / 200

Exam Title : GS Simulator Test 5


Email

Exam URL : http://compete.forumias.com

11.

Consider the following characteristics of a soil.


1. High water retention capacity
2. Deficient in Nitrogen and Phosphorous
3. Self-Ploughing
Which of the following soil type has above characteristics?
a) Black Soil
b) Alluvial Soil
c) Laterite Soil
d) Red and Yellow Soil
Correct Answer: A
Your Answer: Unanswered
Explanation:
(a) Black soil / regur soil:
Regur means cotton best soil for cotton cultivation.
Most of the Deccan is occupied by Black soil.
It has high water retaining capacity.
Swells and will become sticky when wet and shrink when dried.
Self-ploughing is a characteristic of the black soil as it develops wide cracks when dried.
Rich in: Iron, lime, calcium, potassium, aluminum and magnesium.
Deficient in: Nitrogen, Phosphorous and organic matter.

12. Which among the following is/are the initiatives to enhance energy efficiency?
1. Market Transformation for Energy Efficiency
2. Energy efficiency financing platform
3. Framework for energy efficient economic development (FEED)
Select the correct answer using the codes given below:
a) 1 and 2 only
b) 2 and 3 only
c) 1 and 3 only
d) 1, 2 and 3
Correct Answer: D
Your Answer: Unanswered
Explanation:

ForumIAS
Web: http://www.forumias.com/
Email: ravi@forumias.com

Page 9

Score: 0.00 / 200

Exam Title : GS Simulator Test 5


Email

Exam URL : http://compete.forumias.com

Explanation 12 :
The National Mission for Enhanced Energy Efficiency (NMEEE) is one of the eight missions under the
National Action Plan on Climate Change (NAPCC). NMEEE aims to strengthen the market for energy
efficiency by creating conducive regulatory and policy regime and has envisaged fostering innovative
and sustainable business models to the energy efficiency sector.
The NMEEE spelt out four initiatives to enhance energy efficiency in energy intensive industries which
are as follows:
1. Perform Achieve and Trade Scheme (PAT), a regulatory instrument to reduce specific energy
consumption in energy intensive industries, with an associated market based mechanism to enhance the
cost effectiveness through certification of excess energy saving which can be traded.
2. Market Transformation for Energy Efficiency (MTEE), for accelerating the shift to energy efficient
appliances in designated sectors through innovative measures to make the products more affordable.
3. Energy Efficiency Financing Platform (EEFP), for creation of mechanisms that would help finance
demand side management programmes in all sectors by capturing future energy savings.
4. Framework for Energy Efficient Economic Development (FEEED), for development of fiscal
instruments to promote energy efficiency.
13. Consider the following statements regarding inflation indices.
1. All the three consumer price indices include certain services.
2. Wholesale price index measures inflation at each stage of production.
3. Real estate and insurance services do not find a mention in either CPI or WPI.
Select the correct statement(s) using the codes given below:
a) a ) 1 and 2 only
b) 1 and 3 only
c) 2 and 3 only
d) 1, 2 and 3
Correct Answer: C
Your Answer: Unanswered
Explanation:

ForumIAS
Web: http://www.forumias.com/
Email: ravi@forumias.com

Page 10

Score: 0.00 / 200

Exam Title : GS Simulator Test 5


Email

Exam URL : http://compete.forumias.com

Explanation 13: Consumer Price Index is a measure of change in retail prices of goods and services
consumed by defined population group in a given area with reference to a base year. This basket of
goods and services represents the level of living or the utility derived by the consumers at given levels of
their income, prices and tastes. The consumer price index number measures changes only in one of the
factors; prices. This index is an important economic indicator and is widely considered as a barometer of
inflation, a tool for monitoring price stability and as a deflator in national accounts.
Wholesale Price Index (WPI) is the index is the most widely used inflation indicator in India. This is
published by the Office of Economic Adviser, Ministry of Commerce and Industry. WPI captures price
movements in a most comprehensive way. It is widely used by Government, banks, industry and
business circles. Important monetary and fiscal policy changes are linked to WPI movements. It is in use
since 1939 and is being published since 1947 regularly.
Some of the major services like trade, hotels, financing, insurance, real estate and business services do
not find mention in CPI or WPI.
14. A shadow government has recently been formed in UK by the opposition party of which of the following
countries?
1. Mauritius
2. Maldives
3. Seychelles
4. Sri Lanka
a) 1 only
b) 2 only
c) 1 and 3 only
d) 2 and 4 only
Correct Answer: A
Your Answer: Unanswered
Explanation:

ForumIAS
Web: http://www.forumias.com/
Email: ravi@forumias.com

Page 11

Score: 0.00 / 200

Exam Title : GS Simulator Test 5


Email

Exam URL : http://compete.forumias.com

Maldives opposition parties have formed a shadow government in London by the name of Maldives
United Opposition.

Background

Mr. Nasheed, the first democratically elected leader of the Maldives was forced out of office in 2012,

four years after his election, and was convicted and jailed on disputed terrorism charges.
Mr. Nasheed, was recently given political asylum in the U.K and he called for unity of the opposition

parties against the current President Yameen.


Vision and tasks of the new government

Its objective is to fight for the restoration of democracy in the Indian Ocean nation.
It has called for the immediate removal of President Abdulla Yameen through all legal and lawful

mechanisms.
It also called for the immediate restoration of freedoms unconstitutionally suspended by the
Maldivian government.
It has also demanded establishing a transparent system to monitor public finances and safeguard
the national treasury and protect the rights of 1700 individuals currently under prosecution,
investigation and arbitrary arrest.
Indian recognition:

The government wants to formally seek recognition and assistance for their cause from India
The government is critical of the growing economic footprint of China in the Maldives, which they
believe President Yameen has encouraged to counter Indias influence in the Indian Ocean region.

15. Which among the following organism(s) can be termed as bio fertilizers?
1. Azotobacter 2. Azolla
3. Blue green algae 4. Mycorrhizal
Select the correct answer using the codes given below:
a) 1, 2 and 4 only
b) 2, 3 and 4 only
c) 1, 2 and 3 only
d) 1, 2, 3 and 4
Correct Answer: D
Your Answer: Unanswered
Explanation:

ForumIAS
Web: http://www.forumias.com/
Email: ravi@forumias.com

Page 12

Score: 0.00 / 200

Exam Title : GS Simulator Test 5


Email

Exam URL : http://compete.forumias.com

(d) Important bio fertilizers


Following are some of the important types of bio fertilizers which can be considered for agro based
industries.
Rhizobium biofertilizer: Rhizobium is a symbiotic bacteria forming root nodules in legume plants.
These nodules act as miniature nitrogen production factories in the fields. The nodule bacteria fix more
nitrogen (N2) than needed by legume plant and the bacteria. The surplus fixed nitrogen is then secreted
and fertilizes the soil. Rhizobium is more efficient than-free living nitrogen-fixing bacteria and can fix upto
200 kg N/ha/yr.
Azotobacter biofertilizer: Azobacter are aerobic free living nitrogen fixers. They grow in the
rhizosphere (around the roots) and fix atmospheric nitrogen non symbiotically and make it available to
the particular cereals. These bacteria produce growth promoting hormones which helps in enhancing
growth and yield of the plant.
Azospirillium biofertilizer: These are aerobic free living nitrogen fixers which live in associative
symbiosis. In this type of association bacteria live on the root surface of the host plant and do not form
any nodule with roots of grasses. It increases crop yield and its inoculation benefits crop. They also
benefit the host plants by supplying growth hormones and vitamins. These bacteria are commonly used
for the preparation of commercial inoculants.
Blue green algae: Blue green algae (BGA or cyanobacteria) like Nostoc and Anabaena are free living
photosynthetic organisms also capable of fixing atmospheric nitrogen. In the flooded rice fields blue
green algae serves as a nitrogen biofertilizer.
Azolla bio fertilizers: Azolla is a water fern inside which grows the nitrogen fixing blue green algae
Anabaena. It contains 2-3% nitrogen when wet and also produces organic matter in the soil. The Azolla
-Anabaena combination type biofertilizer is used all over the world. This can be grown in a cooler
regions. But there is a need to develop a strain that can tolerant to high temperature, salinity and
resistant to pests and diseases. Production technology is very easy and can be adopted by rice farmers.
The only constraint in Azolla is that it is an aquatic plant and water becomes limiting factor in growing it
particularly in summer.
Phosphorus solubilising biofertilizer: Phosphorus is an important element required for plant growth.
This element is also needed for nodulation by rhizobium. Some microorganisms are capable of
solubilizing immobilized phosphorus making it available to plants for absorption.
Mycorrhizal fungi acts as biofertilizer are known to occur naturally on roots of forest trees and crop
plants. In soils low in available nutrients there is an increased absorption of nutrients by plants infected
with Mycorrhiza. The fungus has the ability to dissolve and absorb phosphorus that plant roots cannot
readily absorb

ForumIAS
Web: http://www.forumias.com/
Email: ravi@forumias.com

Page 13

Score: 0.00 / 200

Exam Title : GS Simulator Test 5


Email

Exam URL : http://compete.forumias.com

16.

Which among the following state(s) has/have bicameral legislature?


1. Bihar 2. West Bengal 3. Madhya Pradesh
4. Jammu and Kashmir 5. Jharkhand
Select the correct answer using the codes given below:
a) 1, 2 and 4 only
b) 1, 3 and 5 only
c) 1 and 4 only
d) 1, 2 and 3 only
Correct Answer: C
Your Answer: Unanswered
Explanation:
Explanation 16 : The Vidhan Parishad (or Legislative Council) is the upper house in those states of India
that have a bicameral legislature. As of 2014, seven (out of twenty-nine) states have a Legislative
Council: Andhra Pradesh, Bihar, Jammu and Kashmir, Karnataka, Maharashtra, Telangana ,and Uttar
Pradesh. In addition, Rajasthan and Assam are cleared by the parliament of India to make their own
Vidhan Parishads .

17. Which of the following were the main objectives of the Khalifat Movement?
1. To rouse anti-British feelings among the Muslims of India.
2. To reform the Muslim society.
3. To demand separate electorates and preserve the Khalifat .
4. To save the Ottoman Empire and preserve the Khalifat
Select the correct answer from the codes given below:
a) a ) 1 and 2 only
b) 2 and 3 only
c) 3 and 4 only
d) 1 and 4 only
Correct Answer: D
Your Answer: Unanswered
Explanation:

ForumIAS
Web: http://www.forumias.com/
Email: ravi@forumias.com

Page 14

Score: 0.00 / 200

Exam Title : GS Simulator Test 5


Email

Exam URL : http://compete.forumias.com

Explanation 17 : The Khilafat movement was a pan-Islamic , political protest campaign launched by
Muslims in British India to influence the British government.Khilafat movement, force that arose in India
in the early 20th century as a result of Muslim fears for the integrity of Islam. These fears were aroused
by Italian and Balkan attacks on Turkeywhose sultan, as caliph, was the religious head of the
worldwide Muslim communityand by Turkish defeats in World War I. They were intensified by the
Treaty of Svres ), which not only detached all non-Turkish regions from the empire but also gave parts
of the Turkish homeland to Greece and other non-Muslim powers.
In India Maulana Mohammad Ali Jauhar and his brother Maulana Shaukat Ali along with some other
Muslim leaders such as Dr. Mukhtar Ahmed Ansari. Raees-ul-Muhajireen Barrister Jan Muhammad
Junejo , Hasrat Mohani , Maulana Abul Kalam Azad and Dr. Hakim Ajmal Khan joined hands a created
an All India Khilafat Committee, at Lucknow.. It had two main demands, which were never accepted:
1.) Caliph Sultan must retain sufficient territories so that he is able to defend the Islamic Faith.
2.) The places which are called Jazirat-ul-arab , including the Arabia, Syria, Iraq and Palestine must
remain under Muslim suzerainty.
18. Under which of the following conditions, the UPSC can serve the needs of a state?
1. When requested by the President
2. When requested by the Governor
3. With the approval of Parliament
4. With the approval of President
5. With the approval of the concerned state legislature.
Select the correct answer using the codes given below:
a) 1 and 3 only
b) 2 and 5 only
c) 2 and 4 only
d) 1 and 5 only
Correct Answer: C
Your Answer: Unanswered
Explanation:

ForumIAS
Web: http://www.forumias.com/
Email: ravi@forumias.com

Page 15

Score: 0.00 / 200

Exam Title : GS Simulator Test 5


Email

Exam URL : http://compete.forumias.com

Explanation 18:
Copyright www.www.examrace.com
Provisions regarding the Union Public Service Commissions are contained from Article 315 to Article
323.
Commission consists of a chairman and ten members. The terms and conditions of service of chairman
and members of the Commission are governed by the Union Public Service Commission (Members)
Regulations, 1969. The Chairman and other members of the UPSC (Union Public Service Commission)
are appointed by the President of India . At least half of the members of the Commission are Civil
Servants (working or retired) with minimum ten years of experience either in Central or State service.
The Commission is serviced by a Secretariat headed by a Secretary with two Additional Secretaries, a
number of Joint Secretaries, Deputy Secretaries and other supporting staff.
Every member holds office for a term of six years or until he attains the age of sixty-five years, whichever
is earlier.
The Constitution makes a provision for the establishment of a Joint State Public Service Commission
(JSPSC) for two or more states. While the UPSC and the SPSC are created directly by the Constitution,
a JSPSC can be created by an act of Parliament on the request of the state legislatures concerned.
Thus, a JSPSC is a statutory and not a constitutional body.
UPSC cans serve the needs of the state on request of the state governor and with the approval of the
President.
19. Consider the following statement regarding island of Lakshadweep.
1. Crescentic reef can be found in the east.
2. Eastern seaboard is steeper.
3. Minicoy island lies north of the 9 degree channel.
Select the correct answer using the codes given below:
a) a ) 1 and 2 only
b) 2 and 3 only
c) 1 and 3 only
d) 1, 2 and 3
Correct Answer: B
Your Answer: Unanswered
Explanation:

ForumIAS
Web: http://www.forumias.com/
Email: ravi@forumias.com

Page 16

Score: 0.00 / 200

Exam Title : GS Simulator Test 5


Email

Exam URL : http://compete.forumias.com

Explanation 19: In the Arabian Sea, roughly 200 kilometers (120 miles) off the west coast of southern
India, are the Lakshadweep Islands. The island chain consists of 27 islands along with coral atolls and
sand banks. Lakshadweeps coral atolls glow iridescent blue-green in this natural- color scene. The
islands and atolls might appear scattered, but they all lie along a prominent north-south submarine ridge
The Lakshadweep ridge rises from a depth of more than 2,000 meters (6,500 feet) below sea level, and
submarine surveys indicate that the ridge is probably steeper on its eastern flank than on its western
flank. At the surface, the coral atolls show consistent differences between their eastern and western
sides. Most of the saucer-shaped lagoons in Lakshadweep are confined to the western sides of the
islands. Reefs exist on the eastern shores, but the shallow, rocky terrain has likely stunted them.
Minicoy island lies to the south of the 9 degree channel.
20. With reference to famous Mohiniattam dance, consider the following statements.
1. It is a centuries old living tradition of Kerala, developed from Devdasi system.
2. Its striking feature is the swaying of the dancer from side to side and the smooth and unbroken flow of
the body movement.
3. Performers usually wear an off-white colored sari with gold brocade borders.
Select the correct statement using the codes given below.
a)
b)
c)
d)

1 only
1 and 2 only
2 and 3 only
1, 2 and 3

Correct Answer: D
Your Answer: Unanswered
Explanation:
(d) Mohiniattam is a classical dance form of Kerala. Mohiniattam is derived from the words " Mohini "
(meaning beautiful women) and " attam "(meaning dance). Thus, Mohiniattam dance form is a beautiful
feminine style with surging flow of body movements. Mohiniattam dance in Kerala developed in the
tradition of Devadasi system, which later grew and developed a classical status.
21. Consider the following rivers of India.
1. Periyar
2. Sabarmati
3. Vaigai
4. Baitarni
Which of the above river(s) is/are west flowing rivers of India?
a) 3 only
b) 1, 2 and 4 only

ForumIAS
Web: http://www.forumias.com/
Email: ravi@forumias.com

Page 17

Score: 0.00 / 200

Exam Title : GS Simulator Test 5


Email

Exam URL : http://compete.forumias.com

c) 1 and 2 only
d) 1, 2 and 3 only
Correct Answer: C
Your Answer: Unanswered
Explanation:
Explanation 21: Vaigai River, river in Tamil Nadu state, southern India, flowing 150 miles (240 km)
generally southeast. Rising in the Varushanad Hills of western Tamil Nadu, it initially flows northeast
through the Kambam and Varushanad valleys. In its central reaches the Vaigai flows eastward into the
Vaigai reservoir at Narasingapuram . Near Sholavandan it bends to the southeast, passing Madurai on
its course to its mouth on Palk Strait.
The Baitarani is one of the important east flowing rivers of peninsular India, flowing eastward and joining
the Bay of Bengal. The river rises in the hill ranges of Keonjhar District of Orissa near Manakarancho
village at an elevation of about 900m above M.S.L. On its way, many tributaries join the river from both
banks. You can read here
Sabarmati and Periyar are west flowing river.
22. Which of the following disease(s) are not communicable by sharing the body piercing needles?
1. Kala azar 2. Dengue
3. Malaria 4. AIDS
Select the correct answer using the codes given below:
a)
b)
c)
d)

2 and 3 only
1 and 3 only
1, 2 and 3 only
1, 2, 3 and 4

Correct Answer: A
Your Answer: Unanswered
Explanation:
Explanation 22: It is HIV that mainly is transmitted via the needle piercing. Rest other are transmitted
through vectors.

ForumIAS
Web: http://www.forumias.com/
Email: ravi@forumias.com

Page 18

Score: 0.00 / 200

Exam Title : GS Simulator Test 5


Email

Exam URL : http://compete.forumias.com

23. Which among the following is/are the arguments against GDP being a reliable source of income
calculation?
1. Exclusion of care economy
2. Unpaid Charitable work
3. Non monetized portion of economy
4. Environmental Cost
Select the correct answer using the codes given below:
a) 1, 2 and 4 only
b) 1, 3 and 4 only
c) 2, 3 and 4 only
d) 1, 2, 3 and 4
Correct Answer: D
Your Answer: Unanswered
Explanation:
Explanation 23 : GDP is the total value of goods and services produced within the country. It is criticized
mainly on the ground that:
1. GDP does not take into account the sustainability of future GDP
2. GDP does not take into account the value of non-monetized activity
3. GDP does not differentiate between more or less productive economic activity .
4. It does not take into account work of housewife.
5. It excludes care economy
6. It excludes unpaid charitable work.
24. Which among the following set of pair(s) are correctly matched?
Multilateral Institution Purpose
1. IMF Promote international financial stability and monetary cooperation
2. Asian Development Bank Develop good governance and fight corruption
3. World Bank Promote long term economic development and poverty reduction
Select the correct answer using the codes given below:
a) 1 and 2 only
b) 2 and 3 only
c) 1 and 3 only
d) 1, 2 and 3
Correct Answer: A

ForumIAS
Web: http://www.forumias.com/
Email: ravi@forumias.com

Page 19

Score: 0.00 / 200

Exam Title : GS Simulator Test 5


Email

Exam URL : http://compete.forumias.com

Your Answer: Unanswered


Explanation:

ForumIAS
Web: http://www.forumias.com/
Email: ravi@forumias.com

Page 20

Score: 0.00 / 200

Exam Title : GS Simulator Test 5


Email

Exam URL : http://compete.forumias.com

Explanation 24
The International Monetary Fund (IMF) is an organization of 189 countries, working to foster global
monetary cooperation, secure financial stability, facilitate international trade, promote high employment
and sustainable economic growth, and reduce poverty around the world.
The IMFs fundamental mission is to ensure the stability of the international monetary system. It does so
in three ways: keeping track of the global economy and the economies of member countries; lending to
countries with balance of payments difficulties; and giving practical help to members.
The IMF oversees the international monetary system and monitors the economic and financial policies of
its 189 member countries. As part of this process, which takes place both at the global level and in
individual countries, the IMF highlights possible risks to stability and advises on needed policy
adjustments.
A core responsibility of the IMF is to provide loans to member countries experiencing actual or potential
balance of payments problems. This financial assistance enables countries to rebuild their international
reserves, stabilize their currencies, continue paying for imports, and restore conditions for strong
economic growth, while undertaking policies to correct underlying problems.
Unlike development banks, the IMF does not lend for specific projects.
The IMF helps its member countries design economic policies and manage their financial affairs more
effectively by strengthening their human and institutional capacity through technical assistance and
training. The IMF aims to exploit synergies between technical assistance and trainingwhich it calls
capacity developmentto maximize their effectiveness.
The Asian Development Bank was conceived in the early 1960s as a financial institution that would be
Asian in character and foster economic growth and cooperation in one of the poorest regions in the
world. Good governance is critical for development. The lack of it hinders the delivery of public services,
promotes corruption, and inhibits economic development.
ADB promotes good governance processes and practices for inclusive participatory, and sustainable
development in Asia and the Pacific.
The World Bank is an international financial institution that provides loans to developing countries for
capital programs. It comprises two institutions: the International Bank for Reconstruction and
Development (IBRD), and the International Development Association (IDA). The World Bank is a
component of the World Bank Group, which is part of the United Nations system.
The World Bank's official goal is the reduction of poverty. However, according to its Articles of
Agreement, all its decisions must be guided by a commitment to the promotion of foreign investment and
international trade and to the facilitation of Capital investment.

ForumIAS
Web: http://www.forumias.com/
Email: ravi@forumias.com

Page 21

Score: 0.00 / 200

Exam Title : GS Simulator Test 5


Email

Exam URL : http://compete.forumias.com

25.

Consider the following regarding sovereign wealth funds.


1. Sovereign wealth funds can be invested domestically only.
2. National Investment and Infrastructure Fund (NIIF) is Indias first sovereign wealth fund.
Select the correct answer using the codes given below:
a) 1 only
b) 2 only
c) Both 1 and 2
d) Neither 1 nor 2
Correct Answer: B
Your Answer: Unanswered
Explanation:
(b) Please refer:
http://forumias.com/portal/9-pm-daily-brief-2-june-2016/#Govt.-mulls-outcome-linked-incentive-for-NIIF-chief

26. Which among the following pairs(s) is/are incorrectly matched?


Industry Solid Waste
1. Sugar Industry Lime Mud
2. Pulp and Paper Industry Press Mud
3. Fertilizer Industry Gypsum
Select the correct answer using the codes given below:
a) 1 and 2 only
b) 2 and 3 only
c) 1 and 3 only
d) 1, 2 and 3
Correct Answer: A
Your Answer: Unanswered
Explanation:

ForumIAS
Web: http://www.forumias.com/
Email: ravi@forumias.com

Page 22

Score: 0.00 / 200

Exam Title : GS Simulator Test 5


Email

Exam URL : http://compete.forumias.com

(a) Industrial solid waste


The major producers of solid waste are:
Thermal power plants producing coal ash/ fly ash;
The integrated iron and steel mills producing blast furnace slag;
Non-ferrous industries like aluminium, copper and zinc producing red mud and tailings;
Sugar industries generating press mud;
Pulp and paper industries producing lime mud;
Fertilizer and allied industries producing gypsum;
Hospitals producing bio-medical waste
27. Consider the following statement regarding FRBMA act:
1. It follows the range deficit target when it comes to targeting the fiscal deficit.
2. It is an act which aims at institutionalizing financial discipline and reducing Indias Fiscal deficit.
3. One of the objectives of the act is to aim for fiscal stability in the shorter run.
4. As per the act, the aim of fiscal deficit reduction is 3.9% of GDP for the year 2015-16.
Which of the statements given above is/are true?
a)
b)
c)
d)

1 and 4 only
1, 2 and 4 only
2 and 4 only
1, 2, 3 and 4

Correct Answer: C
Your Answer: Unanswered
Explanation:
(d) FRBMA act follows point deficit targeting method instead of range deficit method. One of the
objectives of the act is to aim for fiscal stability in the longer run and not in the shorter run. Rest all the
statements are true.
28. Consider the following statements regarding Public Interest Litigation (PIL).
1. PIL is defined by an act of parliament.
2. PIL does away with the concept of locus standi.
3. PIL was derived based on Article 21 and 14 of fundamental rights.
Select the correct answer using the codes given below:
a) 1 and 2 only

ForumIAS
Web: http://www.forumias.com/
Email: ravi@forumias.com

Page 23

Score: 0.00 / 200

Exam Title : GS Simulator Test 5


Email

Exam URL : http://compete.forumias.com

b) 2 and 3 only
c) 1 and 3 only
d) 1, 2 and 3
Correct Answer: B
Your Answer: Unanswered
Explanation:

ForumIAS
Web: http://www.forumias.com/
Email: ravi@forumias.com

Page 24

Score: 0.00 / 200

Exam Title : GS Simulator Test 5


Email

Exam URL : http://compete.forumias.com

Explanation 28 : A Public Interest Litigation (PIL) can be filed in any High Court or directly in the Supreme Court. It is not
necessary that the petitioner has suffered some injury of his own or has had personal grievance to litigate. PIL is a right
given to the socially conscious member or a public spirited NGO to espouse a public cause by seeking judicial for
redressal of public injury. Such injury may arise from breach of public duty or due to a violation of some provision of the
Constitution. Public interest litigation is the device by which public participation in judicial review of administrative action is
assured. It has the effect of making judicial process little more democratic.
According to the guidelines of the Supreme Court any member of public having sufficient interest may maintain an action
or petition by way of PIL provided: 1. There is a personal injury or injury to a disadvantaged section of the population for whom access to legal justice system
is difficult,
2. The person bringing the action has sufficient interest to maintain an action of public injury,
3. The injury must have arisen because of breach of public duty or violation of the Constitution or of the law, It must seek
enforcement of such public duty and observance of the constitutional law or legal provisions.
4. This is a powerful safeguard and has provided immense social benefits, where there is essentially failure on the part of
the execute to ameliorate the problems of the oppressed citizens.
Among the numerous factors that have contributed to the growth of PIL in this country, the following deserve special
mention:
1. The character of the Indian Constitution. Unlike Britain, India has a written constitution which through Part III
(Fundamental Rights) and Part IV (Directive Principles of State Policy) provides a framework for regulating relations
between the state and its citizens and between citizens inter-se.
2. India has some of the most progressive social legislation to be found anywhere in the world whether it be relating to
bonded labor , minimum wages, land ceiling, environmental protection, etc. This has made it easier for the courts to haul
up the executive when it is not performing its duties in ensuring the rights of the poor as per the law of the land.
3. The liberal interpretation of locus standi where any person can apply to the court on behalf of those who are
economically or physically unable to come before it has helped. Judges themselves have in some cases initiated suo moto
action based on newspaper articles or letters received
4. Although social and economic rights given in the Indian Constitution under Part IV are not legally enforceable, courts
have creatively read these into fundamental rights thereby making them judicially enforceable. For instance the "right to
life" in Article 21 has been expanded to include right to free legal aid, right to live with dignity, right to education, right to
work, freedom from torture, barfetters and hand cuffing in prisons, etc.
5. Sensitive judges have constantly innovated on the side of the poor. for instance, in the Bandhua Mukti Morcha case in
1983, the Supreme Court put the burden of proof on the respondent stating it would treat every case of forced labor as a
case of bonded labor unless proven otherwise by the employer. Similarly in the Asiad workers judgment case, Justice P.N.
Bhagwati held that anyone getting less than the minimum wage can approach the Supreme Court directly without going
through the labor commissioner and lower courts
6. In PIL cases where the petitioner is not in a position to provide all the necessary evidence, either because it is
voluminous or because the parties are weak socially or economically, courts have appointed commissions to collect
information on facts and present it before the bench.

ForumIAS
Web: http://www.forumias.com/
Email: ravi@forumias.com

Page 25

Score: 0.00 / 200

Exam Title : GS Simulator Test 5


Email

Exam URL : http://compete.forumias.com

29.

Consider the following statements.


1. India was to become a federation if more than 50% of the Princely states decide to join it.
2. The ministers of the provincial government were to be responsible to the legislature.
3. Diarchy was introduced at the provincial level.
Which of the above statements was/were the provisions of Government of India Act, 1935?
a) 1 and 2 only
b) 2 and 3 only
c) 1 and 3 only
d) 1, 2 and 3
Correct Answer: A
Your Answer: Unanswered
Explanation:
(a) The main provisions of the Government of India Act, 1935 are
1. India was to become a federation if more than 50% of the princely states decide to join it.
2. Provincial Autonomy was granted. The ministers of provincial govt. were to be responsible to the
legislature. The power of the legislature was also increased.
3. Diarchy was abolished at the provincial level but introduced at the centre. The viceroy became more
powerful and was not to be responsible to the legislature.

30. Which among the following is/are the performance index to calculate MGNREGA index?
1. Average days of employment per household.
2. Percentage wages paid within the promised 1 month of enlisting to work
3. The work completion rate
Select the correct answer using the codes given below:
a)
b)
c)
d)

1 and 2 only
2 and 3 only
1 and 3 only
1, 2 and 3

Correct Answer: C
Your Answer: Unanswered
Explanation:
(c) Please refer: http://forumias.com/portal/9-pm-daily-brief-31-may-2016/#The-MGNREGA-index

ForumIAS
Web: http://www.forumias.com/
Email: ravi@forumias.com

Page 26

Score: 0.00 / 200

Exam Title : GS Simulator Test 5


Email

Exam URL : http://compete.forumias.com

31. Consider the following statements about Vedanta philosophy and select the correct ones
1. The theory of Karma is linked to the Vedanta Philosophy.
2. According to this philosophy, if a person acquires the knowledge of the self ( atma ) he realizes the
knowledge of Brahma, and thus attains salvation.
3. Purusha and Prakriti are two main elements associated with Vedanata .
Select the correct answer using the codes given below:
a) 1 and 2 only
b) 1 and 3 only
c) 2 and 3 only
d) 1, 2 and 3
Correct Answer: A
Your Answer: Unanswered
Explanation:
(a) Purusha and Prakriti are two main elements associated with Samkhya philosophy.
32. Consider the following international agreements:
1. Millennium Development Goals
2. The world Heritage Convention
3. Cartagena Protocol
Which of the above has/have a bearing on the biodiversity and climate change?
a) 3 only
b) 1 and 3 only
c) 2 and 3 only
d) 1, 2 and 3
Correct Answer: D
Your Answer: Unanswered
Explanation:

ForumIAS
Web: http://www.forumias.com/
Email: ravi@forumias.com

Page 27

Score: 0.00 / 200

Exam Title : GS Simulator Test 5


Email

Exam URL : http://compete.forumias.com

(d) Goal 7 of the MDGs is to ensure environmental sustainability. One target (target 2) specifically
concerns biodiversity. The target is to reduce biodiversity loss, achieving, by 2010, a significant reduction
in the rate of loss.
UNESCO's World Heritage mission is to encourage countries to sign the World Heritage Convention and
to ensure the protection of their natural and cultural heritage.
The Cartagena Protocol on Bio-safety: On 29 January 2000, the Conference of the Parties to the
Convention on Biological Diversity adopted a supplementary agreement to the Convention known as the
Cartagena Protocol on Bio-safety. The Protocol seeks to protect biological diversity from the potential
risks posed by living modified organisms resulting from modern biotechnology. It establishes an
advanced informed agreement (AIA) procedure for ensuring that countries are provided with the
information necessary to make informed decisions before agreeing to the import of such organisms into
their territory.
33. Termination of a sitting of the House without any definite date being fixed for the next sitting is termed as
a)
b)
c)
d)

Adjournment sine die


Adjournment
Guillotine
None of the above

Correct Answer: A
Your Answer: Unanswered
Explanation:
(a) Adjournment sine die (from the Latin "without day") means "without assigning a day for a further
meeting or hearing". To adjourn an assembly sine die is to adjourn it for an indefinite period.
34. Consider the following statements
1. India has made a contribution to 1mn USD to the Nuclear Security Fund
2. The first Nuclear Security Summit was held in Seoul in 2010
3. The 4th Nuclear Security Summit saw the participation of all recognised nuclear states.
a) 1 only
b) 2 only
c) 1 and 3 only
d) 1, 2 and 3
Correct Answer: A
Your Answer: Unanswered
Explanation:

ForumIAS
Web: http://www.forumias.com/
Email: ravi@forumias.com

Page 28

Score: 0.00 / 200

Exam Title : GS Simulator Test 5


Email

Exam URL : http://compete.forumias.com

Nuclear Security Summit (NSS), an initiative of President Barack Obama to coordinate international
efforts to prevent terror organizations from acquiring nuclear weapons or material. The first Nuclear
Security Summit was held in Washington, DC in 2010, and was followed by Summits in Seoul in 2012
and The Hague in 2014.
There are twin goals for the 2016 Nuclear Security Summit: 1) advancing tangible improvements in
nuclear security behavior 2) Strengthening the global nuclear security architecture.
India has played an active role in these summits. India made a voluntary contribution of one million
dollars to the Nuclear Security Fund.
35. In reference to the tax scenario of the Indian economy, consider the following statements:
1. Over the year, direct tax base has been widening.
2. Revenue from Indirect taxes has been increasing in comparison to direct taxes.
3. Wider indirect taxation is better in comparison to wider direct taxation.
Which of the statements given above is/are true?
a)
b)
c)
d)

a ) 1 and 2 only
2 and 3 only
2 only
1, 2 and 3

Correct Answer: C
Your Answer: Unanswered
Explanation:
(c) Over the years, direct tax base has been narrowing. Wider direct taxation is better than indirect
taxation because Indirect taxes directly affect the poor. Statement 2 is correct.
36. Which of the following is not a condition of admissibility of cut motions in the Parliament?
a) It should not make suggestions for the amendment of existing laws.
b) It should not relate to expenditure charged on the Consolidated Fund of India.
c) It should not raise a question of privilege.
d) It should relate to more than one demand.
Correct Answer: D
Your Answer: Unanswered
Explanation:

ForumIAS
Web: http://www.forumias.com/
Email: ravi@forumias.com

Page 29

Score: 0.00 / 200

Exam Title : GS Simulator Test 5


Email

Exam URL : http://compete.forumias.com

Explanation 36 : As per Parliament of India website:


Cut motions are motions that are moved to reduce the amount of a demand in any of the following ways:(a) 'that the amount of the demand be reduced to Re.1/-' representing disapproval of the policy underlying the demand. Such a
motion shall be known as 'Disapproval of Policy Cut'. A member giving notice of such a motion shall indicate in precise terms the
particulars of the policy which he proposes to discuss. The discussion shall be confined to the specific point or points mentioned in
the notice and it shall be open to members to advocate an alternative policy;
(b) ' that the amount of the demand be reduced by a specified amount' representing the economy that can be effected. Such
specified amount may be either a lump sum reduction in the demand or omission or reduction of an item in the demand. The
motion shall be known as 'Economy Cut'. The notice shall indicate briefly and precisely the particular matter on which discussion is
sought to be raised and speeches shall be confined to the discussion as to how economy can be effected;
(c) 'that the amount of the demand be reduced by Rs.100/-' in order to ventilate a specific grievance which is within the sphere of
the responsibility of the Government of India. Such a motion shall be known as 'Token Cut' and the discussion thereon shall be
confined to the particular grievance specified in the motion.
Admissibility of cut motions
210. In order that notice of motion for reduction of the amount of demand may be admissible, it shall satisfy the following
conditions, namely:(I) it shall relate to one demand only;
(ii) it shall be clearly expressed and shall not contain arguments, inferences, ironical expressions, imputations, epithets or
defamatory statements;
(iii) it shall be confined to one specific matter which shall be stated in precise terms;
(iv) it shall not reflect on the character or conduct of any person whose conduct can only be challenged on a substantive motion;
(v) it shall not make suggestions for the amendment or repeal of existing laws;
(vi) it shall not refer to a matter which is not primarily the concern of the Government of India;
(vii) it shall not relate to expenditure charged on the Consolidated Fund of India;
(viii) it shall not relate to a matter which is under adjudication by a court of law having jurisdiction in any part of India;
(ix) it shall not raise a question of privilege;
(x) it shall not revive discussion on a matter which has been discussed in the same session and on which a decision has been
taken;
(xi) it shall not anticipate a matter which has been previously appointed for consideration in the same session;
(xii) it shall not ordinarily seek to raise a discussion on a matter pending before any statutory tribunal or statutory authority
performing any judicial or quasi-judicial functions or any commission or court of enquiry appointed to enquire into, or investigate
any matter:
Provided that the Speaker may in his discretion allow such matter being raised in the House as is concerned with the procedure or
stage of enquiry, if the Speaker is satisfied that it is not likely to prejudice the consideration of such matter by the statutory tribunal,
statutory authority, commission or court of enquiry; and
(xiii) it shall not relate to a trivial matter.

ForumIAS
Web: http://www.forumias.com/
Email: ravi@forumias.com

Page 30

Score: 0.00 / 200

Exam Title : GS Simulator Test 5


Email

Exam URL : http://compete.forumias.com

37.

Which among the following state(s) do( es ) not have any national park?
1. Punjab 2. Haryana
3. Jharkhand 4. Tripura
Select the correct answer using the codes given below:
a) 2 and 3 only
b) 1 and 4 only
c) 1 only
d) 4 only
Correct Answer: C
Your Answer: Unanswered
Explanation:
(c)

38. Consider the following statements in the context of humidity:


1. There is no unit of absolute humidity. It is expressed in percentage.
2. Relative humidity is expressed in gram per cubic metre.
3. Increase in the temperature of air does not affect the potency of humidity.
Which of the above statement(s) is/are in correct?
a)
b)
c)
d)

1 only
2 only
3 only
1, 2 and 3

Correct Answer: D
Your Answer: Unanswered
Explanation:
Explanation 38: A xerocole commonly referred to as a desert animal, is an animal adapted to live in the
desert. The main challenges they must overcome are lack of water and excessive heat. To conserve
water, they both avoid evaporation and concentrate excretions (i.e. urine and feces ). Some are so adept
at conserving water or obtaining it from food that they do not need to drink at all. To escape the desert
heat, xerocoles tend to be either nocturnal or crepuscular, most active at dawn and dusk.
Hydrocoles are Aquatic animals which live in and require large quantities of water.
Mesocoles are Terrestrial animals requiring moderate amounts of water.
39. The 1916 Session of Indian National Congress is of significance in the history of the Freedom Movement
because
a) a joint session with Muslim League was held to sign the historic Lucknow pact

ForumIAS
Web: http://www.forumias.com/
Email: ravi@forumias.com

Page 31

Score: 0.00 / 200

Exam Title : GS Simulator Test 5


Email

Exam URL : http://compete.forumias.com

b) attainment of Pooran Swaraj was adopted as the goal of Congress


c) resolution against Indian participation in the World War was adopted
d) decision to boycott the round table conference was taken
Correct Answer: A
Your Answer: Unanswered
Explanation:

ForumIAS
Web: http://www.forumias.com/
Email: ravi@forumias.com

Page 32

Score: 0.00 / 200

Exam Title : GS Simulator Test 5


Email

Exam URL : http://compete.forumias.com

The 1916 session was know for coming together of Congress and Muslim League for the so called
illusory Hindu Muslim Unity.
Congress and Muslim league negotiated and reached an agreement to pressure the British government
to adopt a more liberal approach to India and give Indians more authority in respect with the self
government.
This was for the first time that Muslim League and Congress met at a common platform for the first time
since their birth. The soul of this pact is written here.

There shall be self-government in India.

Muslims should be given one-third representation in the central government.


There should be separate electorates for all the communities until a community demanded joint

electorates.
A system of weightage to minority political representation (giving minorities more representation in
the government then is proportional to their share of the population) should be adopted.
The number of the members of Central Legislative Council should be increased to 150. At the
provincial level, four-fifth of the members of the Legislative Councils should be elected and one-fifth
should be nominated.
The size of provincial legislatures should not be less than 125 in the major provinces and from 50 to
75 in the minor provinces. All members, except those nominated, should be elected directly on the
basis of adult franchise.
No bill concerning a community should be passed if the bill is opposed by three-fourth of the
members of that community in the Legislative Council. The term of the Legislative Council should be
five years.
Members of Legislative Council should themselves elect their president. Half of the members of
Imperial Legislative Council should be Indians. The salaries of the Secretary of State for Indian
Affairs should be paid by the British government and not from Indian funds.
Of the two Under Secretaries, one should be Indian.
The Executive should be separated from the Judiciary.
At that time it was called a symbol of Hindu Muslim unity and Sarojini Nayudu hailed Jinnah as an
Ambassador of Hindu-Muslim Unity.
The unity last a few years. Till Lucknow pact, Muslim League was nowhere in the national politics.
By making this pact, the Congress recognized that it is a political party that represents the Muslims of
India.
This was a mistake, this was a wrong belief that showed its results very soon.
40. Animals which can tolerate extremely dry conditions and pass long periods without water are termed as

ForumIAS
Web: http://www.forumias.com/
Email: ravi@forumias.com

Page 33

Score: 0.00 / 200

Exam Title : GS Simulator Test 5


Email

Exam URL : http://compete.forumias.com

a) Hydrocoles
b) Xerocoles
c) Mesocoles
d) None of the above
Correct Answer: B
Your Answer: Unanswered
Explanation:
(b)
41. With reference to the Indian history of art and culture, consider the following pairs:
Famous Piece of Art

Place of Location/Site

1. A huge monolithic statue of Bahubali


A. Ellora
2. Sculpture of Ravan shaking Mount Kailasha
B. Gomasteshwara
3. Rock cut Panch Rathas Monument
C. Kancheepuram
Match the above correctly and choose the correct codes from below
a) 1 - B , 2 - A, 3 - C
b) 1 - A , 2 - B, 3 - C
c) 1 - C, 2 - A, 3 - B
d) 1 - A, 2 - C, 3 - B
Correct Answer: A
Your Answer: Unanswered
Explanation:

ForumIAS
Web: http://www.forumias.com/
Email: ravi@forumias.com

Page 34

Score: 0.00 / 200

Exam Title : GS Simulator Test 5


Email

Exam URL : http://compete.forumias.com

The huge status of Bahubali is located at Gomateshwara . Bahubali is a much revered figure among
Jains. He is said to have meditated motionless for a whole year in kayotsarga (standing) posture
because of which climbers grew around his legs. After one year of meditation, Bahubali attained Kevala
Jnana (omniscience) and became an omniscient being. According to Jain texts, Bahubali's soul attained
moksha (liberation from the cycle of births and deaths) at Mount Kailash. He is revered as a Siddha
(liberated soul) by the Jains. Bahubali is also called Gommatesha because of the Gommateshwara
statue dedicated to him.
The statue built by the Ganga dynasty minister and commander Chavundaraya, is a 57-foot (17 m)
monolith (statue carved from a single piece of rock) and is situated above a hill in Shravanabelagola, in
the Hassan district of Karnataka state, India. It was built in around 983 A.D. and is one of the largest free
standing statues in the world.
The theme of Ravana shaking Mount Kailasha has been depicted several times in the caves of Ellora.
The most noteworthy of all is depicted on the left wall of Kailashnath temple at Ellora.
Pancha Rathas (also known as Pandava Rathas) is a monument complex at Mahabalipuram, on the
Coromandel Coast of the Bay of Bengal, in the Kancheepuram district of the state of Tamil Nadu, India.
Pancha Rathas is an example of monolithic Indian rock-cut architecture.
Dating from the late 7th century, it is attributed to the reign of King Mahendravarman I and his son
Narasimhavarman I (630680 AD; also called Mamalla, or "great warrior") of the Pallava Kingdom. An
innovation of Narasimhavarman, the structures are without any precedent in Indian architecture. The
complex is under the auspices of the Archaeological Survey of India (ASI) and is part of the UNESCO
World Heritage site inscribed by UNESCO as Group of Monuments at Mahabalipuram.
42. Consider the following statements about Heart of Asia conference which was recently held in New
Delhi:
1. Its objective was to bringing peace and stability to South China Sea region.
2. China and Pakistan did not took participation in the conference.
Select the correct answer using the codes given below:
a) 1 only
b) 2 only
c) Both 1 and 2
d) Neither 1 nor 2
Correct Answer: D
Your Answer: Unanswered
Explanation:

ForumIAS
Web: http://www.forumias.com/
Email: ravi@forumias.com

Page 35

Score: 0.00 / 200

Exam Title : GS Simulator Test 5


Email

Exam URL : http://compete.forumias.com

(d) The Heart of Asia ( HoA ) conference was held in New Delhi with the objective of bringing peace and
stability to Afghanistan.
14 participating countries: Russia, China, India, Pakistan, Afghanistan, Iran, Kazakhstan, Kyrgyz
Republic, Tajikistan, Turkmenistan, Saudi Arabia, UAE, Azerbaijan and Turkey
43. Consider the following pairs
1. Vembad Kol Wetland

A. Man made Wetland

2. Harike Lake Wetland


3. Loktak Lake

B. Largest wetland of India


C. Largest Freshwater lake in North East

Which of the above pairs are matched correctly?


a) 1 and 2 only
b) 3 only
c) 1 and 3 only
d) 1, 2 and 3
Correct Answer: B
Your Answer: Unanswered
Explanation:

ForumIAS
Web: http://www.forumias.com/
Email: ravi@forumias.com

Page 36

Score: 0.00 / 200

Exam Title : GS Simulator Test 5


Email

Exam URL : http://compete.forumias.com

Only 3 is correct.
Loktak Lake is the largest freshwater lake in North -East India is famous for the phumdis (heterogeneous mass of
vegetation, soil, and organic matters at various stages of decomposition) floating over it.
India became a contracting party to the Ramsar Convention in October 1981 and designated Chilika Lake (Odisha)
and Keoladeo National Park (Rajasthan) as its first two Ramsar Sites. Four additional sites were designated in
1990: Sambhar Lake (Rajasthan), Loktak Lake (Manipur), Harike Lake (Punjab) and Wular Lake (Jammu &
Kashmir). Currently, India has 26 Ramsar Sites as follows:
Andhra Pradesh Kolleru Lake Assam Deepor Beel
Himachal Pradesh Chandertal Wetland Pong Dam Lake Renuka Wetland (This is smallest wetland of India)
Jammu & Kashmir Hokera Wetland Surinsar-Mansar Lakes Tsomoriri Wular Lake
Kerala Ashtamudi Wetland Sasthamkotta Lake Vembanad-Kol Wetland (Largest Wetland of India)
Madhya Pradesh Bhoj Wetland Manipur Loktak Lake (Montreux Record) Odisha Bhitarkanika Mangroves Chilika
Lake
Punjab Harike Lake (Harike Wetland and the lake are manmade and were formed by constructing the head works
across the Sutlej river, in 1953) Kanjli Ropar Rajasthan Keoladeo National Park (Montreux Record) Sambhar Lake
Tamil Nadu Point Calimere Wildlife and Bird Sanctuary Tripura Rudrasagar Lake
Uttar Pradesh Upper Ganga River (Brijghat to Narora Stretch) West Bengal East Calcutta Wetlands Gujarat
Nalsarovar Bird Sanctuary (Latest wetland added)
Currently, two wetlands of India are in Montreux record viz. Keoladeo National Park, Rajasthan and Loktak Lake,
Manipur. Further, Chilka lake was placed in the record but was later removed from it.
About Montreaux Record (Important )
If the ecological character of any Ramsar wetland has changed, or is changing or is likely to change as the result of
technological developments, pollution or other human interference, it will inform without delay to the Ramsar
Secretariat.
Once this information has been provided to Ramsar Secretariat, it will do the following
1) Enter the wetland into its Montreux Record (a record for such sites where there has been or likely to be adverse
ecological change)
2) Send a Ramsar Advisory Mission to the country.
3) This mission will analyse the situation and define how to tackle the threats to the wetland. Once the appropriate
measures have been taken, the site will be removed from Montreux Record.

ForumIAS
Web: http://www.forumias.com/
Email: ravi@forumias.com

Page 37

Score: 0.00 / 200

Exam Title : GS Simulator Test 5


Email

Exam URL : http://compete.forumias.com

44.

Which of the following statements are correct regarding the powers of a Governor to reserve a bill for the
consideration of the President?
1. It is laid down in Article 200.
2. It is not a discretionary power of the Governor.
3. It is compulsory, if the bill endangers the position of High Court.
4. He can reserve any bill passed by the state legislature.
Select the correct answer using the codes given below
a) 1, 2 and 3 only
b) 3 and 4 only
c) 1 and 3 only
d) 2, 3 and 4 only
Correct Answer: C
Your Answer: Unanswered
Explanation:
Explanation 44 : According to Article 200, when a Bill passed by the Legislature of a State is presented
to the Governor, he has four options, namely,
(a) He assents to the Bill;
(b) He withholds assent;
(c) He reserves the Bill for the consideration of the President; or
(d) He returns the Bill to the Legislature for reconsideration.
The first proviso says that as soon as the Bill is presented to him, he may return the Bill to the
Legislature (if it is not a Money Bill) together with a message requesting the Legislature to reconsider the
Bill. He can also suggest the desirability of introducing such amendments or changes as he thinks
appropriate. If, on such reconsideration, the Bill is passed again, with or without amendments, and is
presented to the Governor for assent, he has to accord his assent. The second proviso says that if the
Bill presented to him derogates, in the opinion of Governor, from the powers of the High Court so as to
endanger the position which the High court is designed to fill by the Constitution, he is bound to reserve
the Bill for the consideration of the President.

ForumIAS
Web: http://www.forumias.com/
Email: ravi@forumias.com

Page 38

Score: 0.00 / 200

Exam Title : GS Simulator Test 5


Email

Exam URL : http://compete.forumias.com

45. Consider the following statement regarding Pradhan Mantri Mudra Yojana :
1. MUDRA is an institution set up up by the govt. to provide funding to farm sector, particularly small and
marginal farmers.
2. Under the Kishor category of Mudra, loan cover of Rs . 50,000 can be availed.
3. Regional Rural Banks (RRBs) are not entitled to provide loans under Mudra.
4. Mudra Bank has been allotted a refinance fund of Rs . 20,000 crores from the shortfall of Priority
Sector Lending.
Which of the statements given above is/are true?
a) 1, 2 and 3 only
b) 1, 2 and 4 only
c) 2 and 3 only
d) 4 only
Correct Answer: D
Your Answer: Unanswered
Explanation:
(d) MUDRA is set up to fund non-farm sectors. Under the Kishor category loans above Rs . 50,000 and
up to Rs 500,000 can be availed. RRBs are entitled to provide loans under Mudra. Statement 4 is
correct.
46. Which among the following mineral(s) are extracted from the ocean using existing technologies?
1. Magnesium 3. Sodium Chloride
2. Bromine 4. Zinc
Select the correct answer using the codes given below:
a) 2 and 3 only
b) 1 and 3 only
c) 3 and 4 only
d) 1, 2 and 3 only
Correct Answer: D
Your Answer: Unanswered
Explanation:

ForumIAS
Web: http://www.forumias.com/
Email: ravi@forumias.com

Page 39

Score: 0.00 / 200

Exam Title : GS Simulator Test 5


Email

Exam URL : http://compete.forumias.com

(d) Ocean mineral resources are found in sea water and on deep ocean floor. High cost of mineral
extraction from sea water, where they occur in low concentration, is not economical. Only magnesium,
bromine and sodium chloride are abundant enough to be extracted profitably using existing technologies.
Manganese rich nodules present on the deep ocean floor may be a future source of manganese and
other important metals. They can be sucked up by giant vacuum pipes by mining ship. But because of
high cost involved in the process and who owns them or the ocean have kept people away from this
project. Rich deposits of gold, silver, zinc and copper are found as sulphide in the deep-ocean floor. But
it costs too much to extract these minerals.
47. Chabahar Port, recently in news is located in the
a) Persian Gulf
b) Gulf of Oman
c) Gulf of Aden
d) Red Sea
Correct Answer: B
Your Answer: Unanswered
Explanation:

ForumIAS
Web: http://www.forumias.com/
Email: ravi@forumias.com

Page 40

Score: 0.00 / 200

Exam Title : GS Simulator Test 5


Email

Exam URL : http://compete.forumias.com

Chabahar Port is located in the Gulf of Oman. About Chabahar Port - from ForumIAS Editorial.
Location In south-eastern Iran, it is located on Makran coast in the Gulf of Oman.
Background In 2003, the Iran and India had agreed to develop the Chabahar port.
Importance of Chabahar port Economic, Political, Diplomatic and Strategic.
Despite the strategic import of Chabahar for India, there has been very little progress on it for several reasons Irans unenthusiastic support for the project.
Development till now India signed trilateral pact for the strategically located Chabahar Port along with Iran and Afghanistan.
What could be done India might be better off building an international consortium to invest in the project.

Location

In south-eastern Iran, it is located on Makran coast in the Gulf of Oman. It is the only Iranian port having direct access to ocean. Though it is a free trade and industrial zone but is underdeveloped especially when compared to the sprawling port of Bandar Abbas further west.

Background

In 2003, the Iran and India had agreed to develop the port but the venture made slow progress due to Western curbs on Iran over its controversial nuclear programme.
But after the sanction has been lifted both side trying to fast-track the plan.

Importance of Chabahar port

Economic

Chabahar port is the part of North-South Transport Corridor, will make India to export the goods to central asia and Europe.
The International NorthSouth Transport Corridor is the ship, rail, and road route for moving freight between India, Russia, Iran, Europe and Central Asia. The route primarily involves moving freight from India, Iran, Azerbaijan and Russia via ship, rail and road.

It would provide an alternative access to trade with Afghanistan bypassing Pakistan.

The free-trade zone around the port can be an important trade centre with West Asia.
Reduced Current Account Deficit: Reduced transportation costs will allow India to import crude oil, urea and dry fruits at lower prices.
Political

It will reduce dependence of land-locked Afghanistan on Pakistan for sea-access lowering Pakistans strategic hold on Afghanistan (as Pakistan denies access to India for trade in Afghanistan via the Wagah Border).
Both Iran and India has stake in the stability of Afghanistan. Chabahar provides India an easier land-sea route to Afghanistan, where it has fostered close security cooperation and economic interests over the years. India has already spent $100 million on building a 220-kilometre road in the Nimroz province of Afghanistan. The
road will be extended to Chabahar. India has pledged $100m for laying railway lines connecting Afghanistan with Central Asia. Increasing trade would help in the stability of Afghanistan.
China is encircling India through its policy of string of pearls. Pursuant to this policy China is developing Gwadar Port of Pakistan. Developing Chabahar port will help to encounter Chinese presence in this region.
It will provide India access to the Middle East and Gulf countries increasing trade and ties.
Provides India a foothold in the western Arabian Sea, which is important as many of our energy imports pass.
Diplomatic

It will provide a link to promote cultural linkages between India and Iran.

It can act as a centre to coordinate humanitarian operations such as mass evacuation of refugees in times of need.
It will provide the diplomatic edge to Indias ambition of developing a blue-water navy.
Strategic

First, it is the nearest port to India on the Iranian coast, which provides access to the resources and markets of Afghanistan and Central Asia.
Second, it is located 76 nautical miles (less than 150km) west of the Pakistani port of Gwadar, being developed by China; this makes it ideal for keeping track of Chinese or Pakistani military activity based out of Gwadar.

Despite the strategic import of Chabahar for India, there has been very little progress on it for several reasons

Irans unenthusiastic support for the project. Although the idea was first mooted in 2003, it was only in 2012 on the sidelines of the 16th Non-Aligned Movement Summit in Tehran that Iran (then reeling under sanctions for its nuclear activities) conceded to set up a joint working group to operationalize the port project as part of the
trilateral cooperation agreement between Afghanistan, India and Iran on investment cooperation, trade and transit.

A key factor behind Irans reluctance to allow an Indian presence at Chabahar was the opposition by the Army of the Guardians of the Islamic Revolution (the so-called Revolutionary Guards), which reportedly uses the port to ship arms to Yemen and militant groups in the region.
Its strategic significance notwithstanding, the economic viability of the project is suspect. India, which has had trouble raising funds for the project, has so far been able to invest only $85 million to build a couple of berths. While India recently indicated that it was willing to invest up to $20 billionone of its largest overseas

venturesto develop the port, petrochemical and fertilizer plants in the Chabahar SEZ, it remains to be seen if it can raise the funds.
Given the presence of Gwadar next door, where China has already invested over $1 billion and committed another $46 billion for the 3,000-km long economic corridor to link Gwadar to Kashgar in Xinjiang province and its One Belt, One Road project, it is unclear whether the Chabahar route will generate enough trade and traffic to

justify the investment.


In fact, Tehran, which has been playing hardball with India and demanding greater Indian investment in Chabahar, itself plans to invest $4 billion to build a refinery in Gwadar to process 400,000 barrels of oil per day.

Development till now

The Union Cabinet has given its approval to the proposal of the Ministry of Shipping for provision and operationalization of credit of 150 million USD from EXIM Bank for development of Chabahar Port in Iran in February 2016.

An MoU was signed between India and Iran. As per the MoU, India is to equip and operate two berths in Chabahar Port Phase-I with capital investment of USD 85.21 million and annual revenue expenditure of USD 22.95 million on a ten year lease. Ownership of equipment will be transferred to Iranian side on completion of 10
year period or for an extended period, based on mutual agreement.

As per the MoU, operation of two berths will commence within a period of maximum 18 months after the signing of the Contract.
The two berths will be operated by the India Ports Global Private Limited, a Company promoted by the Jawaharlal Nehru Port Trust and Kandla Port Trust two major ports working under the Ministry of Shipping.

The Union Cabinet has now authorized the Ministers of Finance, External Affairs and Shipping to approve the final contract with Iran and for resolution of any issue arising in implementation of the project.
The Union Cabinet has also authorized the Ministry of Shipping to form a Company in Iran for implementing the Chabahar Port Development Project and related activities.

Indian Prime Minister visited Iran in May 2016 and signed trilateral pact for the strategically located Chabahar Port along with Iran and Afghanistan.

What could be done

Resolving the Chabahar conundrum is vital to securing Indias interests in Iran and beyond.

According to some experts given the challenges in manifesting this project, India is unlikely to succeed on its own. India might be better off building an international consortium with the likes of Japan, Australia and South Korea to invest in the project.
Japan also needs Central Asian markets and would like to export there and would like to have first mover advantage once the port develops. Also Australia also has a stake in the route as they see a lot of their exports passing through Chabahar.

ForumIAS
Web: http://www.forumias.com/
Email: ravi@forumias.com

Page 41

Score: 0.00 / 200

Exam Title : GS Simulator Test 5


Email

Exam URL : http://compete.forumias.com

48. Consider the following statements about Acharya J. B. Kriplani .


1. He was a partisan socialist.
2. He held the presidency of INC during the turbulent period of 1947.
3. He was the supporter of Gandhian Philosophy.
Select the correct statement using the codes given below.
a) a ) 1 and 2 only
b) 1 and 3 only
c) 2 only
d) 1, 2 and 3 only
Correct Answer: D
Your Answer: Unanswered
Explanation:
(d) A passionate freedom fighter, a partisan socialist, a Gandhian by Heart, and a competent teacher;
these are the terms that are often relate d tot the name that is Acharya Jivatram Bhagwandas Kriplani .
Although most remembered for holding the presidency of the Indian National Congress during the
turbulent period of 1947, his contribution is much, much more.
49. Which among the following forest(s) is/are most immune to forest fires?
1. Tropical Wet Forests
2. Temperate Montane Forests
Select the correct answer using the codes given below:
a)
b)
c)
d)

1 only
2 only
Both 1 and 2
Neither 1 nor 2

Correct Answer: C
Your Answer: Unanswered
Explanation:
(c)
50. Consider the following statements.
1. In protected forests, hunting, grazing, etc. are banned unless specific orders are issued otherwise.
2. In reserved forests, hunting and grazing are sometimes allowed for communities living on the fringes
of the forest.
Select the correct statement(s) using the codes given below.
a) 1 only
b) 2 only

ForumIAS
Web: http://www.forumias.com/
Email: ravi@forumias.com

Page 42

Score: 0.00 / 200

Exam Title : GS Simulator Test 5


Email

Exam URL : http://compete.forumias.com

c) Both 1 and 2
d) None of the above
Correct Answer: D
Your Answer: Unanswered
Explanation:
(d) In this question, definition has been interchanged. So, both the statements are incorrect.
Rights to all activities like hunting, grazing, etc. in reserved forests are banned unless specific orders are
issued otherwise. In protected areas, rights to activities like hunting and grazing are sometimes given to
communities living on the fringes of the forest, who sustain their livelihood partially or wholly from forest
resources or products. The first reserve forest in India was Satpura National Park in Madhya Pradesh.
51. If in an election to a state legislative assembly, the candidate who is declared elected loses his deposit, it
means that
a)
b)
c)
d)

The polling was very poor


The election was for a multi-member constituency
The elected candidates victory over his nearest rival was very marginal
A very large number of candidates contested the election

Correct Answer: D
Your Answer: Unanswered
Explanation:
(d) A candidate loses his deposit only if he is polled less than 1/6th of votes. This is possible when very
large number of candidates contested the elections.
52. Which of the following statements) is/are correct?
1. Inland Waterways Authority of India was established in October 1986 for development of Inland
waterways.
2. Headquarter of this Authority is in Patna.
3. Bhanga Lakhimpur Waterway is Indias National Waterway 5.
Select the correct answer using the codes given below:
a) 1 only
b) 1 and 2 only
c) 2 and 3 only
d) 1, 2 and 3
Correct Answer: A
Your Answer: Unanswered
Explanation:

ForumIAS
Web: http://www.forumias.com/
Email: ravi@forumias.com

Page 43

Score: 0.00 / 200

Exam Title : GS Simulator Test 5


Email

Exam URL : http://compete.forumias.com

(a) Headquarter of Inland Waterways Authority of India is in Noida. Bhanga Lakhimpur is National
Waterway No. 6. National Waterway No. 5 is Talchar to Paradip
53. Consider the following statements about India-US relations
1. US Congress has introduced a resolution to bring India on a par with Americas NATO allies in terms
of Defence trade and technology transfer
2. A Bill was introduced in the US Congress to help India join the Asia Pacific Economic Cooperation
(APEC).
Select the correct answer using the codes given below:
a) 1 only
b) 2 only
c) Both 1 and 2
d) Neither 1 nor 2
Correct Answer: C
Your Answer: Unanswered
Explanation:
(c) A Congressional resolution aiming to bring India on a par with Americas NATO (North Atlantic Treaty
Organization) allies in terms of Defence trade and technology transfer was introduced recently.
The US-India Defence Technology and Partnership Act (HR 4825) proposes to amends the Arms Export
Control Action.
A group of influential US lawmakers have introduced a Bill in the Congress asking the US President
Barack Obama administration to help India join the Asia Pacific Economic Cooperation (APEC).
54. What happens when the office of both the Speaker and Deputy Speaker of Lok Sabha falls vacant?
a)
b)
c)
d)

The members of Lok Sabha immediately elect a Speaker.


The senior most willing member of Lok Sabha becomes the speaker.
The President appoints any member of Lok Sabha as speaker.
The Deputy Chairman of Rajya Sabha presides over till the next speaker is elected.

Correct Answer: C
Your Answer: Unanswered
Explanation:
(c) In case of vacancy in speaker as well as deputy speaker, President appoints any member of the Lok
Sabha as speaker.

ForumIAS
Web: http://www.forumias.com/
Email: ravi@forumias.com

Page 44

Score: 0.00 / 200

Exam Title : GS Simulator Test 5


Email

Exam URL : http://compete.forumias.com

55. Which of the following pair(s) is/are incorrectly matched?


Sacred Groves State
1. Bamboo Groves North Bengal
2. Orans Chattisgarh
3. Sarnas Rajasthan
4. Kovil Kadu Tamil Nadu
Select the correct answer using the codes given below:
a) 1 and 4 only
b) 1, 2 and 3 only
c) 2 and 3 only
d) 1 and 3 only
Correct Answer: C
Your Answer: Unanswered
Explanation:

ForumIAS
Web: http://www.forumias.com/
Email: ravi@forumias.com

Page 45

Score: 0.00 / 200

Exam Title : GS Simulator Test 5


Email

Exam URL : http://compete.forumias.com

Q ,55 ) Explanation 55: Sacred groves of India are forest fragments of varying sizes, which are communally protected, and which usually have a
significant religious connotation for the protecting community. Hunting and logging are usually strictly prohibited within these patches. Other
forms of forest usage like honey collection and deadwood collection are sometimes allowed on a sustainable basis. Sacred groves did not enjoy
protection via federal legislation in India.
State
Andhra Pradesh

Local name
Pavitraskhetralu

Arunachal Pradesh

Gumpa forests

Assam

Than , Madaico
Sarna , Devlas ,

Chhattisgarh
Mandar , Budhadev
Haryana
Himachal Pradesh

Beed or Bid ( ), Bani ( ), Bann (


), Janglat ( ), Shamlat ( )
Deo bhumi

Jharkhand

Sarna
Devarakadu ,

Karnataka
Devkad
Kerala
Maharashtra
Manipur

Kavu , Sarpa Kavu


Deorai / Devrai
Gamkhap , Mauhak
Law kyntang ,

Meghalaya
Law Lyngdoh
Orissa
Puducherry

Jahera , Thakuramma
Kovil Kadu
Oran Kenkri (Ajmer),
Vani ( Mewar ),

Rajasthan
Shamlat deh , Devbani
( Alwar ), Jogmaya
Gumpa forests
Sikkim

(since attached
to monasteries)

Tamil Nadu

Kovil Kadu

Telangana
Uttarakhand

ForumIAS
Web: http://www.forumias.com/
Email: ravi@forumias.com

Devbhumi , Bugyal

Page 46

Score: 0.00 / 200

Exam Title : GS Simulator Test 5


Email

Exam URL : http://compete.forumias.com

56.

Consider the following statements regarding the objectives of Buffer Stock:


1. Buffer stocks are required to provide for TPDS and other welfare schemes.
2. It ensures food security when the production is short of normal demand.
3. It stabilizes prices during period of production shortfalls.
4. Buffer stock can be used to dispose off the surplus food grins in open market.
Which of the statements given above is/are false?
a) 1, 2 and 4 only
b) 2 and 3 only
c) 3 only
d) 4 only
Correct Answer: D
Your Answer: Unanswered
Explanation:
(d). Statement 4 is not an objective of Buffer stock. Rest all the statements are true.

57. Consider the following statements regarding the recently held World Conference on Tourism for
Development (WCTD).
1. Its first meet was held in New Delhi in 2016
2. The theme of the meet was Tourism for Peace and Development to advance the contribution of
tourism to the UN Sustainable Development Goals (SDG).
Select the correct answer using the codes given below:
a)
b)
c)
d)

1 only
2 only
Both 1 and 2
Neither 1 nor 2

Correct Answer: B
Your Answer: Unanswered
Explanation:
Explanation 57 : The First World Conference on Tourism for Development was held in Beijing, China
from 18-21 May 2016 with a specific view to advancing the contribution of tourism to the UN Sustainable
Development Goals (SDG) under the theme Tourism for Peace and Development.

ForumIAS
Web: http://www.forumias.com/
Email: ravi@forumias.com

Page 47

Score: 0.00 / 200

Exam Title : GS Simulator Test 5


Email

Exam URL : http://compete.forumias.com

58. The recommendations of the Kelkar Committee on Revisiting & Revitalising the PPP Model of
Infrastructure Development include
1. Amendment of Prevention of Corruption Act to differentiate between maladfide and genuine errors by
public servants.
2. Setting up independent regulators in sectors having Public Private Partnerships
3. Complete removal of any scope for renegotiation of infra structure projects of long duration at a later
stage.
a) 1 only
b) 1 and 2 only
c) 2 and 3 only
d) 1, 2 and 3 only
Correct Answer: B
Your Answer: Unanswered
Explanation:
The Kelkar Committee has recommended both 1 . Amendment of Prevention of Corruption Act to
differentiate between maladfide and genuine errors by public servants.
and (2). Setting up independent regulators in sectors having Public Private Partnerships.
It has not completely recommended removing later stage renegotiations , but has come up with
guidelines for negotiations.
59. With reference to UDAY scheme, Consider the following statements.
1. It will improve the operational efficiency of discoms .
2. It will reduce cost of production by rationalizing coal linkages.
3. Future automatic takeover of 50% losses of discoms .
Select the correct statement using the codes given below.
a) a ) 1 and 2 only
b) 2 and 3 only
c) 1 and 3 only
d) 1, 2 and 3
Correct Answer: D
Your Answer: Unanswered
Explanation:

ForumIAS
Web: http://www.forumias.com/
Email: ravi@forumias.com

Page 48

Score: 0.00 / 200

Exam Title : GS Simulator Test 5


Email

Exam URL : http://compete.forumias.com

(d) UDAY Scheme:


1) Improving operational efficiency like through proper metering, using energy efficient equipments
2) Reducing costs by rationalizing coal linkages, improved coal output etc.
3) Reducing interest costs of discoms by partial takeover of debt by state and leftover be reissued at
lower interest.
4) Limit on future debt by discoms by limiting it to 25% of their revenue and for working capital
requirement only.
5) Future automatic takeover of 50% losses of discoms .
60. Which among the following species is/are protected under the crocodile breeding and management
project?
1. Fresh water crocodile
2. Salt water crocodile
3. Gharial
Select the correct answer using the codes given below:
a)
b)
c)
d)

1 only
3 only
1 and 3 only
1, 2 and 3

Correct Answer: D
Your Answer: Unanswered
Explanation:
(d) Crocodile breeding and management project
Started in 1976 with assistance from FAO UNDP to save three endangered species of crocodile,
namely the fresh water crocodile, the salt water crocodile and rare gharial. Sixteen centres of
conservation are there while eleven sanctuaries have been declared specially for crocodile protection
including the National Chambal Sanctuary in Madhya Pradesh.
61. Consider the statements regarding the Ordinance making power of President:
1. The President cannot promulgate an ordinance to amend tax laws.
2. The President cannot promulgate an ordinance to amend the constitution.
3. Ordinance made by the President cannot be reviewed by any court.
4. The maximum life of an ordinance is 6 months.
Which of the statements given above is/are true?

ForumIAS
Web: http://www.forumias.com/
Email: ravi@forumias.com

Page 49

Score: 0.00 / 200

Exam Title : GS Simulator Test 5


Email

Exam URL : http://compete.forumias.com

a) 1, 3 and 4 only
b) 2 only
c) 2 and 4 only
d) 2, 3 and 4 only
Correct Answer: B
Your Answer: Unanswered
Explanation:
(b) President can promulgate an ordinance to amend the tax laws. Ordinance made ny the president can
be questioned by the courts on the grounds of malafide. Maximum life of an ordinance is 6 months and 6
weeks.
62. Which one of the following is INCORRECT in respect of Parliamentary control over the Budget?
a)
b)
c)
d)

Parliament has no say in the preparation of the budget


Parliament has the power to increase expenditure charged on the Consolidated Fund
Parliament has no power to impose a tax without the Presidents recommendation
Parliament has no power to increase a tax without the Presidents recommendation

Correct Answer: B
Your Answer: Unanswered
Explanation:
Explanation 62: Lok Sabha has the power to assent to or refuse to give assent to any Demand or even
to reduce the amount of Grant sought by Government. In Rajya Sabha there is only a General
Discussion on the Budget. It does not vote on the Demands for Grants. Only so much of the amount is
subject to the vote of Lok Sabha as is not a "charged" expenditure on the Consolidated Fund of India.
The "charged" expenditure includes the emoluments of the President and the salaries and allowances of
the Chairman and Deputy Chairman of Rajya Sabha and the Speaker and Deputy Speaker of Lok
Sabha, Judges of Supreme Court, Comptroller and Auditor General of India and certain other items
specified in the Constitution of India. Discussion in Lok Sabha on charged expenditure is permissible
but such expenditure is not voted by the House. Members have full opportunity to criticise the budgetary
provisions during the course of discussion as also to make suggestions for improving the financial
position of the country.
63. Consider the following statements about Mauryan administration.
1. The Samharta was the Chief Tax collector.
2. Panyadhaksha was Head of Trade and Commerce.
3. Sannidhata was the chief custodian of the state treasury and store-house.
Select the correct statement(s) using the codes given below:
a) a ) 1 and 2 only
b) 1 and 3 only

ForumIAS
Web: http://www.forumias.com/
Email: ravi@forumias.com

Page 50

Score: 0.00 / 200

Exam Title : GS Simulator Test 5


Email

Exam URL : http://compete.forumias.com

c) 2 and 3 only
d) 1, 2 and 3
Correct Answer: D
Your Answer: Unanswered
Explanation:
Explanation 63 : All statements are true and factual. The job of the assesor and collector who was known
as Samharta . The chief custodian of the state treasury was known as Sannidhata . Statement 2 is also
correct. You may refer to old NCERT to know more about it.
64. Which of the following are the limitations on the sovereignty of Indian Parliament?
1. Fundamental Rights
2. Judicial Review
3. Federalism
4. Written Constitution
Select the correct answer using the codes given below:
a) 1, 3 and 4 only
b) 1, 2 and 3 only
c) 1 and 2 only
d) 1, 2, 3 and 4 only
Correct Answer: D
Your Answer: Unanswered
Explanation:

ForumIAS
Web: http://www.forumias.com/
Email: ravi@forumias.com

Page 51

Score: 0.00 / 200

Exam Title : GS Simulator Test 5


Email

Exam URL : http://compete.forumias.com

Explanation 64: The factors that limit the sovereignty of Indian Parliament are:
1. Written Nature of the Constitution
The Constitution is the fundamental law of the land in our country. It has defined the authority and jurisdiction of all
the three organs of the Union government and the nature of interrelationship between them. Hence, the Parliament
has to operate within the limits prescribed by the Constitution. There is also a legal distinction between the
legislative authority and the constituent authority of the Parliament. Moreover, to effect certain amendments to the
Constitution, the ratification of half of the states is also required. In Britain, on the other hand, the Constitution is
neither written nor there is anything like a fundamental law of the land.
2. Federal System of Government
India has a federal system of government with a constitutional division of powers between the Union and the states.
Both have to operate within the spheres allotted to them. Hence, the law-making authority of the Parliament gets
confined to the subjects enumerated in the Union List and Concurrent List and does not extend to the subjects
enumerated in the State List (except in five abnormal circumstances and that too for a short period). Britain, on the
other hand, has a unitary system of government and hence, all the powers are vested in the Centre.
3. System of Judicial Review
The adoption of an independent Judiciary with the power of judicial review also restricts the supremacy of our
Parliament. Both the Supreme Court and high courts can declare the laws enacted by the Parliament as void and
ultra vires (unconstitutional), if they contravene any provision of the Constitution. On the other hand, there is no
system of judicial review in Britain. The British Courts have to apply the Parliamentary laws to specific cases,
without examining their constitutionality, legality or reasonableness.
4. Fundamental Rights
The authority of the Parliament is also restricted by the incorporation of a code of justiciable fundamental rights
under Part HI of the Constitution. Article 13 prohibits the State from making a law that either takes away totally or
abrogates in part a fundamental right. Hence, a Parliamentary law that contravenes the fundamental rights shall be
void. In Britain, on the other hand, there is no codification of justiciable fundamental rights in the C ./ institution. The
British Parliament has also not made any law that lays down the fundamental rights of the citizens. However, it does
not mean that the British citizens do not have rights. Though there is no charter guaranteeing rights, there is
maximum liberty in Britain due to the existence of the Rule of Law. Therefore, even though the nomenclature and
organisational pattern of our Parliament is similar to that of the British Parliament, there is a substantial difference
between the two. The Indian Parliament is not a sovereign body in the sense in which the British Parliament is a
sovereign body. Unlike the British Parliament, the authority and jurisdiction of the Indian Parliament are defined,
limited and restrained. In this regard, the Indian Parliament is similar to the American Legislature (known as
Congress). In USA also, the sovereignty of Congress is legally restricted by the written character of the Constitution,
the federal system of government, the system of judicial review and the Bill of Rights.

ForumIAS
Web: http://www.forumias.com/
Email: ravi@forumias.com

Page 52

Score: 0.00 / 200

Exam Title : GS Simulator Test 5


Email

Exam URL : http://compete.forumias.com

65. Consider the following statements in the context of alluvial soil in India:
1. Alluvial Soil generally has higher quantity of potash and lesser quantity of phosphorus.
2. Quantity of sand present in alluvial soil decreases as one moves from west to east.
Which of the following statements) is/are correct?
a) 1 only
b) 2 only
c) Both 1 and 2
d) Neither 1 nor 2
Correct Answer: C
Your Answer: Unanswered
Explanation:
Explanation 65 :A lluvial soils are formed by the depositional work of rivers in the river valleys, flood
plains and deltas. The Sutlej-Ganga plain has the largest deposition of alluvial soils. These are very
fertile soils and this plain is considered to be one of the world's most fertile areas. The deltas of the
Mahanadi, the Godavari, the Krishna and the Cauvary also abound in alluvial soils. Alluvial soils are also
found in Assam Valley, Gujarat plains and the Western Coastal Plain. About 40% area is covered by
alluvial soils.
The physical properties of alluvial soils are determined by climate and vegetation.
These soils vary in nature from sandy loam to clay. They are generally right in potash but poor in
phosphorus. In the Upper and Middle Ganga plain, two different types of alluvial soils have developed,
viz. Khaddar and Bhangar . Khaddar-is the new alluvium and is deposited by floods annually, which
enriches the soil by depositing fine silts. Bhangar represents a system of older alluvium, deposited away
from the flood plains.
Both khaddar and bhangar contain calcareous concretions ( kankars ) which are used for white washing
the-houses in the rural .areas. These days, it forms an important raw material for comment industry.
These soils have 'lime, potash in abundance but are deficient in phosphoric acid and nitrogenous and
organic contents. These soils are more loamy and clayey in the lower and middle Ganga plain and the
Brahmaputra valley. The sand content decreases from the west to east. The color of the alluvial soils
varies from the 'light grey to ashram grey. Its shades depend on the depth of the deposition, the texture
of 'the materials, and the time taken for attaining maturity. Alluvial .soils is very useful for growing a
variety of crops especially cereals and pulses. Besides some commercial crops like cotton, sugarcane
and jute are also grown. As such these soils are most intensively cultivated.

ForumIAS
Web: http://www.forumias.com/
Email: ravi@forumias.com

Page 53

Score: 0.00 / 200

Exam Title : GS Simulator Test 5


Email

Exam URL : http://compete.forumias.com

66. Recently, the Indian government has given its approval for the Ashgabat Agreement. Which of the
following are true about it?
1. The Agreement has Oman, Iran, Turkmenistan, Kazakhstan and Uzbekistan as founding members.
2. It will help in the implementation of the International North South Transport Corridor.
3. It will help India use the existing transport corridor to reach the Eurasia region.
a) 1 only
b) 1 and 2 only
c) 1 , 2 and 3
d) 2 and 3 only
Correct Answer: D
Your Answer: Unanswered
Explanation:
The Union Cabinet has approved to accede to the Ashgabat Agreement, an international transport and
transit corridor facilitating transportation of goods between Central Asia and the Persian Gulf.
The Ashgabat Agreement has Oman, Iran, Turkmenistan and Uzbekistan as founding members.
Kazakhstan has also joined this arrangement subsequently. Accession to the Agreement would enable
India to utilise this existing transport and transit corridor to facilitate trade and commercial interaction with
the Eurasian region.
Further, this would synchronise with our efforts to implement the International North South Transport
Corridor (INSTC) for enhanced connectivity. Indias intention to accede to the Ashgabat Agreement
would now be conveyed to the Depository State (Turkmenistan). India would become party to the
Agreement upon consent of the founding members.
Source: PIB Release - http://pib.nic.in/newsite/PrintRelease.aspx?relid=138309
67. Which of the below countries are not landlocked?
1. Cambodia
2. Laos
3. Bhutan
4. Afghanistan
5. Turkmenistan
a) 1 only
b) 1 and 5 only
c) 2, 3 and 4 only
d) 1, 2 and 5 only
Correct Answer: B

ForumIAS
Web: http://www.forumias.com/
Email: ravi@forumias.com

Page 54

Score: 0.00 / 200

Exam Title : GS Simulator Test 5


Email

Exam URL : http://compete.forumias.com

Your Answer: Unanswered


Explanation:

ForumIAS
Web: http://www.forumias.com/
Email: ravi@forumias.com

Page 55

Score: 0.00 / 200

Exam Title : GS Simulator Test 5


Email

Exam URL : http://compete.forumias.com

Mentor Note: Please take a close look the the countries of South East Asia and Central Asia. This is crucial for both Prelims
and Mains.
Cambodia and Turmenistan are not land locked. The rest are. Turkmenistan has access to Caspian Sea.
Also refer the map given below

ForumIAS
Web: http://www.forumias.com/
Email: ravi@forumias.com

Page 56

Score: 0.00 / 200

Exam Title : GS Simulator Test 5


Email

Exam URL : http://compete.forumias.com

68.

Consider the following statements.


1. Coal-bed methane is the pure methane gas absorbed into solid matrix coal.
2. Oil Shale gas is a mixture of propane and butane only.
Which of the statements given above is/are correct?
a) 1 only
b) 2 only
c) Both 1 and 2
d) None of the above
Correct Answer: A
Your Answer: Unanswered
Explanation:
(a) Coalbed methane (CBM or coal-bed methane), coalbed gas, coal seam gas (CSG), or coal-mine
methane (CMM) is a form of natural gas extracted from coal beds.
There is no exact formula of oil shale gas. Compositions of oil shale gas depends of retorted oil shale
and exploited technology. Typical components of oil shale gas are usually methane, hydrogen, carbon
monoxide, carbon dioxide, nitrogen, and different hydrocarbons like ethylene. It may also consist of
hydrogen sulfide and other impurities

69. Which of the following are true of Central Vigilance Commission?


1. It was set up on the recommendation of Santhanam Committee.
2. It does not exercise superintendence over the functioning of CBI.
3. It is set up by an executive resolution of the Government of India.
4. It consists of a chairman and three members.
Select the correct answers using the codes given below:
a) 1, 2 and 4 only
b) 1, 3 and 4 only
c) 1 and 3 only
d) 1 and 4 only
Correct Answer: C
Your Answer: Unanswered
Explanation:

ForumIAS
Web: http://www.forumias.com/
Email: ravi@forumias.com

Page 57

Score: 0.00 / 200

Exam Title : GS Simulator Test 5


Email

Exam URL : http://compete.forumias.com

Explanation 69 : The Central Vigilance Commission was set up by the Government in February ,1964 on the recommendations of the
Committee on Prevention of Corruption, headed by Shri K. Santhanam , to advise and guide Central Government agencies in the field of
vigilance. CVC is conceived to be the apex vigilance institution, free of control from any executive authority, monitoring all vigilance activity
under the Central Government and advising various authorities in Central Government organizations in planning, executing, reviewing and
reforming their vigilance work.
Consequent upon promulgation of an Ordinance by the President, the Central Vigilance Commission has been made a multi member
Commission with "statutory status" with effect from 25th August ,1998 . The CVC Bill was passed by both the houses of Parliament in 2003
and the President gave its assent on september 11, 2003. Thus the Central Vigilance Commission Act 2003 (No45 0f 2003) came into effect
from that date.
The Commission shall consist of:
1. A Central Vigilance Commissioner - Chairperson;
2. Not more than two Vigilance Commissioners - Members;
Functions and powers of the Central Vigilance Commission under the Central Vigilance Commission Act, 2003
1. Exercise superintendence over the functioning of the Delhi Special Police Establishment (CBI) insofar as it relates to the investigation of
offences under the Prevention of Corruption Act, 1988; or an offence under the Cr.PC for certain categories of public servants section
8(1)(a);
2. Give directions to the Delhi Special Police Establishment (CBI) for superintendence insofar as it relates to the investigation of offences
under the Prevention of Corruption Act, 1988 section 8(1 )( b);
3. To inquire or cause an inquiry or investigation to be made on a reference by the Central Government section 8(1 )( c);
4. To inquire or cause an inquiry or investigation to be made into any complaint received against any official belonging to such category of
officials specified in sub-section 2 of Section 8 of the CVC Act, 2003 section 8(1)(d);
5. Review the progress of investigations conducted by the DSPE into offences alleged to have been committed under the Prevention of
Corruption Act, 1988 or an offence under the Cr.PC section (8)(1)(e);
6. Review the progress of the applications pending with the competent authorities for sanction of prosecution under the Prevention of
Corruption Act, 1988 section 8(1 )( f);
7. Tender advice to the Central Government and its organizations on such matters as may be referred to it by them section 8(1) (g);
8. Exercise superintendence over the vigilance administrations of the various Central Government Ministries, Departments and
Organizations of the Central Government section 8(1 )( h);
9. Shall have all the powers of a Civil court while conducting any inquiry section 11;
10. Respond to Central Government on mandatory consultation with the Commission before making any rules or regulations governing the
vigilance or disciplinary matters relating to the persons appointed to the public services and posts in connection with the affairs of the Union
or to members of the All India Services section 19.
11. The Central Vigilance Commissioner (CVC) is the Chairperson and the Vigilance Commissioners (Members) of the Committee, on
whose recommendations, the Central Government appoints the Director of Enforcement section 25.
12. The Committee concerned with the appointment of the Director of Enforcement is also empowered to recommend, after consultation
with the Director of Enforcement appointment of officers to the posts of the level of Deputy Director and above in the Directorate of
Enforcement section 25;
13. The Central Vigilance Commissioner (CVC) is also the Chairperson and the Vigilance Commissioners (Members) of the Committee
empowered to recommend after consultation with Director (CBI), appointment of officers to the post of the level of SP and above except
Director and also recommend the extension or curtailment of tenure of such officers in the DSPE (CBI) - Section 26 and Section 4C of
DSPE Act, 1946.

ForumIAS
Web: http://www.forumias.com/
Email: ravi@forumias.com

Page 58

Score: 0.00 / 200

Exam Title : GS Simulator Test 5


Email

Exam URL : http://compete.forumias.com

70.

Consider the following characteristics of a Biome.


1. Low biotic diversity
2. Limitation of drainage
3. Short season of growth and reproduction
4. Energy and nutrients in the form of dead organic material.
Which of the following Biomes have above characteristics?
a) Savannah Biome
b) Tundra Biome
c) Tropical Rainforest
d) Temperate Grassland
Correct Answer: B
Your Answer: Unanswered
Explanation:
(b) Characteristics of tundra include:
Extremely cold climate.
Low biotic diversity.
Simple vegetation structure.
Limitation of drainage.
Short season of growth and reproduction.
Energy and nutrients in the form of dead organic material.
Large population oscillations.

71. Consider the following statements regarding Mansabdari system


1. It was introduced by Akbar.
2. Mansabdari meant given ranks for military purposes only.
Select the correct statement(s) using the codes given below:
a) 1 only
b) 2 only
c) Both 1 and 2
d) Neither 1 nor 2
Correct Answer: A
Your Answer: Unanswered
Explanation:

ForumIAS
Web: http://www.forumias.com/
Email: ravi@forumias.com

Page 59

Score: 0.00 / 200

Exam Title : GS Simulator Test 5


Email

Exam URL : http://compete.forumias.com

(a) The term Mansab indicates the rank of its holder. Mansabdari was both civil and military. Almost the
whole nobility, bureaucracy and military holds mansabs .
72. Consider the following rivers:
1. Lohit
2. Subansiri
3. Barak
Which of the above flows/flow through Arunachal Pradesh?
a) 1 only
b) 1 and 2 only
c) 3 only
d) 1, 2 and 3
Correct Answer: B
Your Answer: Unanswered
Explanation:
(d) Lohit and Subansiri are tributaries of Brahmaputra river and flow through the Arunachal Pradesh.
Barak River also flows in Assam.
73. Recently, the government has constituted the N K Singh Committee. Which of the following is true about
it?
1. It is set up to review and propose amendments to the Fiscal Responsibility & Budget Management
Act.
2. Its purpose will be to propose an acceptable fiscal deficit range instead of a fixed fiscal deficit target
number.
a) 1 only
b) 2 only
c) Both 1 & 2
d) None of the Above
Correct Answer: C
Your Answer: Unanswered
Explanation:

ForumIAS
Web: http://www.forumias.com/
Email: ravi@forumias.com

Page 60

Score: 0.00 / 200

Exam Title : GS Simulator Test 5


Email

Exam URL : http://compete.forumias.com

The finance ministryhas constituted a five-member panel, headed by former secretary in the finance
ministry and N.K. Singh, to suggest a road map for fiscal consolidation by 31 October. The other
members of the committee are former finance secretary Sumit Bose, chief economic adviser in the
finance ministry Arvind Subramanian, Reserve Bank of India deputy governor Urjit Patel and National
Institute of Public Finance and Policy director Rathin Roy.
As per the Budget speech The FRBM Act has been under implementation for more than a decade. Both central and state
governments have made significant gains from the implementation of this Act. There is now a school of
thought which believes that instead of fixed numbers as fiscal deficit targets, it may be better to have a
fiscal deficit range as the target, which would give necessary policy space to the government to deal with
dynamic situations. There is also a suggestion that fiscal expansion or contraction should be aligned with
credit contraction or expansion, respectively, in the economy,
74. Consider the following statements regarding the pardoning power of the President:
1. Remission means reducing the period of sentence.
2. Commutation means changing one form of punishment to another.
3. President cannot exercise his pardoning power without the advice by the cabinet.
Select the correct statement(s) using the codes given below:
a)
b)
c)
d)

a ) 1 and 2
1 and 3
2 and 3
1, 2 and 3

Correct Answer: D
Your Answer: Unanswered
Explanation:
(d) All the statements are true.
75. The financial instruments through which an Indian entity can raise money from overseas market in the
rupee is termed as
a) Indian Bonds
b) Masala Bonds
c) Zero Coupon Bonds
d) Emerging market bonds
Correct Answer: B
Your Answer: Unanswered
Explanation:

ForumIAS
Web: http://www.forumias.com/
Email: ravi@forumias.com

Page 61

Score: 0.00 / 200

Exam Title : GS Simulator Test 5


Email

Exam URL : http://compete.forumias.com

Explanation 75 : Masala bond is a term is used to refer to rupee-denominated borrowings by Indian


entities in overseas markets. The International Finance Corporation (IFC), the investment arm of the
World Bank, issued a 1,000 crore bond to fund infrastructure projects in India. These bonds were listed
on the London Stock Exchange (LSE). IFC then named them Masala bonds to give a local flavour by
calling to mind Indian culture and cuisine. While it may seem odd to name a staid debt instrument after
food stuffs, it has been done in the past. Chinese bonds, named Dim-sum bonds after a popular dish in
Hong Kong, have been around for while . So have Japanese bonds named Samurai after the countrys
warrior class.
These are issued to foreign investors and settled in US dollars. Hence the currency risk lies with the
investor and not the issuer, unlike external commercial borrowings (ECBs), where Indian companies
raise money in foreign currency loans. While ECBs help companies take advantage of the lower interest
rates in international markets, the cost of hedging the currency risk can be significant. If unhedged,
adverse exchange rate movements can come back to bite the borrower. But in the case of Masala
bonds, the cost of borrowing can work out much lower.
76. Which of the following statements) is/are correct in the context of Isothermal lines?
1. Isothermal lines mostly follow latitudinal lines in North-South direction.
2. Where there is maximum temperature difference due to dissimilarity in water and land, Isothermal
lines are straight there.
Select the correct answer using the codes given below:
a)
b)
c)
d)

1 only
2 only
Both 1 and 2
Neither 1 nor 2

Correct Answer: D
Your Answer: Unanswered
Explanation:
(d) Isothermal lines mostly follow latitudal lines in East-West direction. The place where water and land
have dissimilarity, Isothermal lines turn there suddenly.
http://geography.about.com/library/misc/blisoline.htm
77. According to a recent report of UN Department of Economic & Social Affair, India has the largest
diaspora in the world. Which of the following are true regarding it?
1. It is the cumulative impact of India's long history of emigration to other countries over two millennia
2. Slavery Abolition Act of 1833 by the British Parliament led to sudden outflow of Indians to other
countries.
3. Lot of Indians migrate for work to oil rich countries in the Gulf for better work opportunities.

ForumIAS
Web: http://www.forumias.com/
Email: ravi@forumias.com

Page 62

Score: 0.00 / 200

Exam Title : GS Simulator Test 5


Email

Exam URL : http://compete.forumias.com

a) 1 and 3 only
b) 3 only
c) 2 and 3 only
d) 1, 2 and 3 only
Correct Answer: C
Your Answer: Unanswered
Explanation:
Indian has been a fertile, warm territory with abundant water , which is why people came to India and not
vice versa for two millennia. It was only after the Advent of Europeans that emigration from India started.
One big reason was abolition of slavery. Since slavery was abolished by British Parliament, there was a
sudden shortage of labour for plantation workers in colonies. It was indentured labour system because of
which a major part of migration of Indians began to countries like Fiji, Mauritius etc.
The migration to Gulf countries also happened with the oil boom because of better employment
opportunities.
78. The Oil for Drugs swap deal recently in news, concerns which of the following countries?
a) Vietnam
b) Venezuela
c) Afghanistan
d) Iran
Correct Answer: B
Your Answer: Unanswered
Explanation:
The country is Venezuela which has high debts to pay to Indian medicine companies. But the collapse of
its socialist economy and fall in its currency makes the repayment impossible.
Several of India's generics producers, led by the country's second-largest player Dr Reddy's
Laboratories Ltd, bet heavily on Venezuela as they sought emerging market alternatives to
slower-growing economies such as the United States.
But the unravelling of Venezuela's socialist economy amid a fall in oil prices has triggered triple-digit
inflation and a full-blown political and financial crisis. Unable to pay its bills, the country is facing severe
shortages of even basic supplies such as food, water and medicines.
Dr Reddy's wrote off $65 million in the March quarter, which it said was almost all the money it was owed
from Venezuela. Rival Glenmark Pharmaceuticals Inc, another major investor, says it is due $45 million.
Like pharmaceutical companies globally - which used to enjoy a preferential exchange rate in Venezuela
- Indian producers have been left badly stung by the collapse of the Bolivar currency.

ForumIAS
Web: http://www.forumias.com/
Email: ravi@forumias.com

Page 63

Score: 0.00 / 200

Exam Title : GS Simulator Test 5


Email

Exam URL : http://compete.forumias.com

79.

Consider the following statements regarding the Nutrient Based Subsidy (NBS) scheme implemented by
government of India:
1. Under NBS, subsidy is given based on the weights of different nutrients (N, P, K and S) in the
fertilizer.
2. Additional subsidy is also provided on fertilizers fortified with secondary and micro nutrients.
3. Urea falls under the ambit of NBS.
4. Under this scheme the maximum retail price of fertilizers has been left open and manufacturers are
allowed to fix the MRP at a reasonable level.
Which of the statements given above is/are true?
a) 1, 2 and 3 only
b) 1, 2 and 4 only
c) 1 and 4 only
d) 1 and 2 only
Correct Answer: B
Your Answer: Unanswered
Explanation:
(b) Urea does not come under NBS. Rest all the statements are true.

80. Consider the following climate and vegetation characteristics.


1. Moderate climate and broad leafed deciduous trees.
2. Uniform precipitation
3. Podozolic soils and have deep profile.
Which kind of forest has the above mentioned characteristics?
a) Boreal Forests
b) Tropical Forests
c) Temperate Forests
d) Subtropical rain forests.
Correct Answer: C
Your Answer: Unanswered
Explanation:

ForumIAS
Web: http://www.forumias.com/
Email: ravi@forumias.com

Page 64

Score: 0.00 / 200

Exam Title : GS Simulator Test 5


Email

Exam URL : http://compete.forumias.com

Explanation 80: The term temperate forest is very broad. It covers the forests found between the
tropical and subtropical regions and the barren, treeless lands of the far north and extreme south.
There are many types of temperate forests, but the main categories are: coniferous forests, mixed
broadleaved/ coniferous forests; and broadleaved forests. Characteristics are:
Temperature varies from -30 C to 30 C.
Precipitation (75-150 cm) is distributed evenly throughout the year.
Soil is fertile, enriched with decaying litter.
Canopy is moderately dense and allows light to penetrate, resulting in well-developed and richly
diversified understory vegetation and stratification of animals.
Flora is characterized by 3-4 tree species per square kilometer . Trees are distinguished by broad
leaves that are lost annually and include such species as oak, hickory, beech, hemlock, maple,
basswood, cottonwood, elm, willow, and spring-flowering herbs.
Fauna is represented by squirrels, rabbits, skunks, birds, deer, mountain lion, bobcat, timber wolf, fox,
and black bear.
Further subdivisions of this group are determined by seasonal distribution of rainfall:
moist conifer and evergreen broad-leaved forests: wet winters and dry summers (rainfall is
concentrated in the winter months and winters are relatively mild).
dry conifer forests: dominate higher elevation zones; low precipitation.
mediterranean forests: precipitation is concentrated in winter, less than 100 cm per year.
temperate coniferous: mild winters, high annual precipitation (greater than 200 cm).
temperate broad-leaved rainforests: mild, frost-free winters, high precipitation (more than 150 cm)
evenly distributed throughout the year.
81. Which among the following factor(s) have prevented India from becoming a manufacturing hub?
1. Favourable terms for MSMEs
2. Deindustrialisation in India due to Chinese imports
3. Emphasis on services sector
4. Lack of ease of doing business
Select the correct answer using the codes given below:
a) 2, 3 and 4 only
b) 1, 2 and 4 only
c) 1, 3 and 4 only

ForumIAS
Web: http://www.forumias.com/
Email: ravi@forumias.com

Page 65

Score: 0.00 / 200

Exam Title : GS Simulator Test 5


Email

Exam URL : http://compete.forumias.com

d) 1, 2, 3 and 4
Correct Answer: D
Your Answer: Unanswered
Explanation:
Explanation 81 : This is basic question of simple economics. All are imp reasons for lack of
manufacturing base in India.
82. With reference to Maternal and neonatal tetanus (MNT), Consider the following statements.
1. It is the consequence of unclean deliveries.
2. India had eradicated maternal and neonatal tetanus in 2015.
Select the correct statement(s) using the codes given below:
a) 1 only
b) 2 only
c) Both 1 and 2
d) None of the above
Correct Answer: C
Your Answer: Unanswered
Explanation:
(c) Maternal and neonatal tetanus (MNT) have been among the most common lethal consequences of
unclean deliveries and umbilical cord care practices. When tetanus develops, mortality rates are
extremely high, especially when appropriate medical care is not available.
This happens despite the fact that MNT deaths can be easily prevented by hygienic delivery and cord
care practices, and/or by immunizing mothers with tetanus vaccine that is cheap and very efficacious.
The MNT Elimination Initiative aims to reduce MNT cases to such low levels that the disease is no longer
a major public health problem. Unlike polio and smallpox, tetanus cannot be eradicated (tetanus spores
are present in the environment worldwide), but through immunization of pregnant women and other
women of reproductive age (WRA) and promotion of more hygienic deliveries and cord care practices,
MNT can be eliminated (defined as less than one case of neonatal tetanus per 1000 live births in every
district).
India had eradicated maternal and neonatal tetanus in 2015. It was announced by Modi
government in August 2015.
83. The Economic Survey has prescribed a 4-D model for the banking sector to face competition in the
changed environment. These 4-D stands for
a) Deregulation, Development, Diversification and Disinvestment
b) Deregulation, Double financial repression, Diversification and Disinvestment

ForumIAS
Web: http://www.forumias.com/
Email: ravi@forumias.com

Page 66

Score: 0.00 / 200

Exam Title : GS Simulator Test 5


Email

Exam URL : http://compete.forumias.com

c) Deregulation, Disinterring, Diversification and Disinvestment


d) Deregulation, Disinterring, Diversification and Differentiation
Correct Answer: D
Your Answer: Unanswered
Explanation:
The four Ds include:
De-regulation (addressing the statutory liquidity ratio (SLR) and priority sector lending (PSL))
Differentiation (within the public sector banks in relation to recapitalisation, shrinking balance sheets, and
ownership)
Diversification (of source of funding within and outside banking)
Disinterring (by improving exit mechanisms).
84. Which of the following statement(s) is/are correct in the context of Stratosphere?
1. Decline in temperature in stratosphere stops with altitude.
2. Temperature remains unchanged upto height of 20 km in this zone.
Select the correct answer using the codes given below:
a)
b)
c)
d)

1 only
2 only
Both 1 and 2
Neither 1 nor 2

Correct Answer: C
Your Answer: Unanswered
Explanation:
(c)

ForumIAS
Web: http://www.forumias.com/
Email: ravi@forumias.com

Page 67

Score: 0.00 / 200

Exam Title : GS Simulator Test 5


Email

Exam URL : http://compete.forumias.com

85.

Consider the following statements regarding New Development Bank (NDB)


1. China, as the largest shareholder enjoys a veto Power in the New Development Bank.
2. The NDB will fund Infrastructure projects in developing Nations
3. The NDB shall have currency reserve arrangements to help countries meet short term liquidity
pressure.
Select the correct answer using the codes given below:
a) a ) 1 and 2 only
b) 2 only
c) 1, 2 and 3
d) 2 and 3 only
Correct Answer: B
Your Answer: Unanswered
Explanation:
(b) About New Development Bank
The New Development Bank is a multilateral bank created by BRICS nations viz. Brazil, Russia, India,
China and South Africa.
It is headquartered in Shanghai, China.
In the NDB, each participant country has been assigned one vote, and none of the countries have veto
power.
Renowned Indian banker Kundapur Vaman Kamath is the President (first) of NDB.
The BRICS bank will start with an initial paid-in-capital of $50 billion. Each BRICS country will contribute
$10 billion. The bank will fund infrastructure projects in developing nations, and will have a $100-billion
currency reserve arrangement (CRA) for helping out countries facing short-term liquidity pressures.
India has conveyed the decision to nominate Mr. Kamath for the BRICS bank presidency to other BRICS
nations Brazil, Russia, China and South Africa. Mr. Kamaths five-year term is to be followed by a
Brazilian and then a Russian.
The BRICS had agreed to set up the $100 billion development bank last year, a step that was at the time
seen as one that could challenge the dominance of the West in the international financial system. The
focus of the international financial system, however, has since then shifted to Chinas initiative, the Asian
Infrastructure Investment Bank (AIIB).
The BRICS nations had also agreed last year that the New Development Bank, which will fund
infrastructure projects in developing nations, would have its headquarters in Shanghai.

ForumIAS
Web: http://www.forumias.com/
Email: ravi@forumias.com

Page 68

Score: 0.00 / 200

Exam Title : GS Simulator Test 5


Email

Exam URL : http://compete.forumias.com

86. With reference to Nirbhaya Missile, which of the following are correct?
1. It is a long range supersonic cruise missile designed by DRDO.
2. It is capable of carrying conventional nuclear warheads.
3. It can deliver 1500 Kgs warhead over a distance of 1000kms.
a) 1 only
b) 1 and 2 only
c) 2 and 3 only
d) 1, 2 and 3
Correct Answer: C
Your Answer: Unanswered
Explanation:
Nirbhay is an all-weather, low-cost, long-range cruise missile capable of carrying conventional and
nuclear war heads. The missile has a range of more than 1000 km, weighs about 1500 kg and has a
length of 6 metres.
The missile is powered by a solid rocket booster for take off which is developed by Advanced Systems
Laboratory (ASL). Upon reaching the required velocity and height, Turbofan engine in the missile takes
over for further propulsion.
The missile is guided by a highly advanced inertial navigation system indigenously developed by
Research Centre Imarat (RCI) and a radio altimeter for the height determination. The two-stage missile
Nirbhay is able to pick out a target and attack it among multiple targets.
The missile has a loitering capability, i.e., it can go round a target and perform several manoeuvres and
then re-engage it.
With two side wings, the missile is capable of flying at different altitudes ranging from 500 m to 4 km
above the ground and can also fly at low altitudes (like low tree level) to avoid detection by enemy radar.
It is capable of delivering 24 different types of warheads depending on mission requirements and uses
an inertial navigation system for guidance.
The missile supplements Brahmos by delivering warheads farther than the 290 km range of Brahmos.
87. Arrange the following stages in coal formation from initial stage to final stage.
1. Peat 2. Anthracite
3. Litminous 4. Lignite
Select the correct answer using the codes given below:
a) 1, 3 , 2, 4
b) 1, 4, 3, 2

ForumIAS
Web: http://www.forumias.com/
Email: ravi@forumias.com

Page 69

Score: 0.00 / 200

Exam Title : GS Simulator Test 5


Email

Exam URL : http://compete.forumias.com

c) 1, 3, 4, 2
d) 1, 4, 2, 3
Correct Answer: B
Your Answer: Unanswered
Explanation:
(b)

88. Which of the following is true regarding a minister belonging to either house of the Parliament?
a)
b)
c)
d)

He can take part in the proceedings as well as vote only in his house.
He can take part as well as vote in any of the houses.
He can vote only in his house but take part in proceedings of the other house
He can vote in the other house but can sit in the proceedings only of his house.

Correct Answer: C
Your Answer: Unanswered
Explanation:
Explanation 88: A minister can take part in proceedings of either house but she can only vote in the
house of which she is a member.
89. The Tropic of Cancer passes through which of the following states?
a) West Bengal, Chhattisgarh, Odisha, Madhya Pradesh
b) Rajasthan, Nagaland, Gujarat, West Bengal
c) Gujarat, Madhya Pradesh, Bihar, West Bengal
d) Mizoram, Tripura, West Bengal, Chhattisgarh
Correct Answer: D
Your Answer: Unanswered
Explanation:

ForumIAS
Web: http://www.forumias.com/
Email: ravi@forumias.com

Page 70

Score: 0.00 / 200

Exam Title : GS Simulator Test 5


Email

Exam URL : http://compete.forumias.com

Tropic of cancer passing through Indian states are as follows


1-Gujarat
2- Rajasthan
3- Madhaya Pradesh
4- Chattisgarh
5- Jharkhand
6- West Bengal
7- Tripura
8- Mizoram
90. With reference to oil shale, consider the following statements.
1. It is also known as Kerogen Shale.
2. Its major deposits have been found in United States.
3. Extraction of shale oil from Oil Shale is cheaper than <g>production</g> of conventional crude oil.
Select the correct statement using the codes given below.
a)
b)
c)
d)
e)

2 only
2 and 3 only
1 and 3 only
1, 2 and 3

Correct Answer: D
Your Answer: Unanswered
Explanation:
(d) Oil shale, also known as kerogen shale, is an organic-rich fine-grained sedimentary rock containing
kerogen (a solid mixture of organic chemical compounds) from which liquid hydrocarbons called shale oil
(not to be confused with tight oilcrude oil occurring naturally in shales) can be produced. Shale oil is a
substitute for conventional crude oil; however, extracting shale oil from oil shale is more costly than the
production of conventional crude oil both financially and in terms of its environmental impact. Deposits of
oil shale occur around the world, including major deposits in the United States.

ForumIAS
Web: http://www.forumias.com/
Email: ravi@forumias.com

Page 71

Score: 0.00 / 200

Exam Title : GS Simulator Test 5


Email

Exam URL : http://compete.forumias.com

91. Consider the following statements regarding Central Cabinet Secretariat.


1. It functions directly under the Prime Minister
2. Cabinet secretary is administrative head of the secretariat.
3. Cabinet Secretary is ex- officio head of civil services board.
Select the correct statement(s) using the codes given below:
a) a ) 1 and 2 only
b) 2 and 3 only
c) 1 and 3 only
d) 1, 2 and 3
Correct Answer: D
Your Answer: Unanswered
Explanation:

ForumIAS
Web: http://www.forumias.com/
Email: ravi@forumias.com

Page 72

Score: 0.00 / 200

Exam Title : GS Simulator Test 5


Email

Exam URL : http://compete.forumias.com

Explanation 91 : Before the adoption of the portfolio system in the Government of India, all governmental business was disposed of by the Governor-General-in Council, the Council
functioning as a joint consultative board. As the amount and complexity of business of the Government increased, the work of the various departments was distributed amongst the
members of the Council only the more important cases being dealt with by the Governor-General or the Council collectively.
This procedure was legalised by the Councils Act of 1861 during the time of Lord Canning, leading to the introduction of the portfolio system and the inception of the Executive Council of
the Governor-General. The Secretariat of the Executive Council was headed by the Private Secretary to the Viceroy, but he did not attend the Council meetings. Lord Willingdon first
started the practice of having his Private Secretary by his side at these meetings. Later, this practice continued and in November, 1935, the Viceroy's Private Secretary was given the
additional designation of Secretary to the Executive Council.
The constitution of the Interim Government in September 1946 brought a change in the name, though little in functions, of this Office. The Executive Council's Secretariat was then
designated as Cabinet Secretariat. It seems, however, at least in retrospect, that Independence brought a sort of change in the functions of the Cabinet Secretariat. It no longer
remained concerned with only the passive work of circulating papers to Ministers and Ministries but developed into an organisation for effecting coordination between the Ministries.
The Cabinet Secretariat is under the direct charge of the Prime Minister. The administrative head of the Secretariat is the Cabinet Secretary who is also the ex-officio Chairman of the
Civil Services Board. In the Government of India (Allocation of Business) Rules, 1961 "Cabinet Secretariat" finds a place in the First Schedule to the Rules. The subjects allotted to this
Secretariat are:1. Secretarial assistance to Cabinet and Cabinet Committees.
2. Rules of Business.
The Cabinet Secretariat is responsible for the administration of the Government of India (Transaction of Business) Rules, 1961 and the Government of India (Allocation of Business)
Rules 1961, facilitating smooth transaction of business in Ministries/Departments of the Government by ensuring adherence to these rules. The Secretariat assists in decision-making in
Government by ensuring Inter-Ministerial coordination, ironing out differences amongst Ministries/Departments and evolving consensus through the instrumentality of the standing/
adhoc Committees of Secretaries. Through this mechanism new policy initiatives are also promoted.
The Cabinet Secretariat ensures that the President, the Vice President and Ministers are kept informed of the major activities of all Ministries/Departments by means of monthly
summary of their activities. Management of major crisis situations in the country and coordinating activities of various Ministries in such a situation is also one of the functions of the
Cabinet Secretariat.

Support to Cabinet Committees:


The secretarial assistance provided by Cabinet Secretariat to the Cabinet and Cabinet committees, includes
1. Convening of the meetings of the Cabinet on the orders of the Prime Minister.
2. Preparation and circulation of the agenda.
3. Circulating papers related to the cases on the agenda.
4. Preparing a record of discussions taken.
5. Circulation of the record after obtaining the approval of the Prime Minister.
6. Watching implementation of the decisions taken by the Cabinet.
The Cabinet Secretariat is the custodian of the papers of the Cabinet meetings.

Promotion of Inter-Ministerial Coordination


Among the inter-Ministerial matters, the coordination is required for:
1. Removing difficulties.
2. Removing differences.
3. Overcoming delays.
4. Coordination in administrative action.
5. Coordination of policies.
While each Ministry is responsible for acting on its own for expeditious implementation of Government policies, plans and programmes, where inter-Ministerial cooperation is involved,
they often seek the assistance of the Cabinet Secretariat. The inter-Ministerial problems are dealt with in the meetings of the Committees of Secretaries (COS). Committees are
constituted for discussing specific matters and proposals emanating from various Secretaries to the Government and meetings are held under the chairmanship of the Cabinet
Secretary. These committees have been able to break bottlenecks or secure mutually supporting inter-Ministerial action.
The discussions of the COS takes place on the basis of a paper formulated by the principal Department concerned and the Department with a different point of view, if any, providing a
supplementary note. The decisions or recommendations of the COS are unanimous. These proceedings are also circulated to and are followed up by the departments. There are other
important functions which it discharges, viz.
1. Monitoring.
2. Coordination.
3. Promoting new policy initiatives.
The Cabinet Secretariat is seen as a useful mechanism by the departments for promoting inter-Ministerial coordination since the Cabinet Secretary is also the head of the civil services.
The Secretaries felt it necessary to keep the Cabinet Secretary informed of developments from time to time. The Transaction of Business Rules also requires them to keep the Cabinet
Secretary informed of developments from time to time, especially if there are any departures from these rules.

ForumIAS
Web: http://www.forumias.com/
Email: ravi@forumias.com

Page 73

Score: 0.00 / 200

Exam Title : GS Simulator Test 5


Email

Exam URL : http://compete.forumias.com

92. Consider the following statements regarding the Bharat Bill Payment System:
1. It is a centralized bill payment system which will allow the user to pay all his repetitive monthly bills
using a single website/outlet.
2. National Payment Corporation has been designated as Bharat Bill Payment Central Unit (BBPCU)
3. It has been modelled on the lines of suggestions given by Umesh Bellur committee.
Which of the statements given above is/are true?
a) a ) 1 and 2 only
b) 1 and 3 only
c) 2 and 3 only
d) 1, 2 and 3
Correct Answer: D
Your Answer: Unanswered
Explanation:
Explanation 92 : Bharat Bill Payment System (BBPS) is an integrated bill payment system which will
offer interoperable bill payment service to customers online as well as through a network of agents on
the ground. The system will provide multiple payment modes and instant confirmation of payment.
Bharat Bill Payment Central Unit (BBPCU) will be a single authorized entity operating the BBPS. The
BBPCU will set necessary operational, technical and business standards for the entire system and its
participants, and also undertake clearing and settlement activities. To start with, the scope of BBPS will
cover repetitive payments for everyday utility services such as electricity, water, gas, telephone and
Direct-to-Home (DTH). Gradually, the scope would be expanded to include other types of repetitive
payments, like school / university fees, municipal taxes etc. Banks and non-bank entities presently
engaged in any of the above bill payment activities falling under the scope of BBPS and desirous of
continuing the activity are mandatorily required to apply for approval / authorisation to Reserve Bank of
India under the Payment and Settlement Systems (PSS) Act 2007.
93. Which of the following works are associated with Tulsidas ?
1. Ramcharitmanas
2. Kavitavali
3. Gitanjali
4. Dasa Bodha
Select the correct answer using the codes given below:
a) a ) 1 and 2
b) 1 and 3
c) 1 and 3
d) 1 and 4
Correct Answer: A
Your Answer: Unanswered

ForumIAS
Web: http://www.forumias.com/
Email: ravi@forumias.com

Page 74

Score: 0.00 / 200

Exam Title : GS Simulator Test 5


Email

Exam URL : http://compete.forumias.com

Explanation:
(a) Tulsidas was the greatest saint-poet of the Ram Bhakti cult of Vaishnavism.He is the celebrated
author of Ramcharitmanas , Kavitavali , and Gitavali . Gitanjali is written by Rabindranath Tagore.
Dasa Bodha is the compilation of writings and sermons of Ramdas .
94. Consider the following statements regarding Trade Facilitation Agreement (TFA)
1. India has formally ratified the TFA.
2. The TFA will enter into force once 50% of members have completed their domestic ratification
process.
3. It was an outcome of WTOs 9th Bali ministerial package of 2013.
Select the correct answer using the codes given below:
a)
b)
c)
d)

1 and 2 only
1 and 3 only
2 and 3 only
1, 2 and 3

Correct Answer: B
Your Answer: Unanswered
Explanation:
(b) The Trade Facilitation Agreement will enter into force once two-thirds of members have completed
their domestic ratification process.
95. When the Couple Protection Rate is going up
a)
b)
c)
d)

Birth rate must necessarily fall


Birth rate will fail only if couples are of younger age
Death rate must necessarily fall
Infant mortality rate must be falling

Correct Answer: A
Your Answer: Unanswered
Explanation:
Explanation 95: You can read about it here
96. Consider the following statements with reference to Attorney General of India:
1. He is appointed by the President.
2. He must have the same qualifications as that required for a Judge of High Court.
3. In the performance of his duties he shall have the right of audience in all courts of India.
Select the correct statements using the codes given below:
a) 1 only

ForumIAS
Web: http://www.forumias.com/
Email: ravi@forumias.com

Page 75

Score: 0.00 / 200

Exam Title : GS Simulator Test 5


Email

Exam URL : http://compete.forumias.com

b) 1 and 3 only
c) 2 and 3 only
d) 1, 2 and 3
Correct Answer: B
Your Answer: Unanswered
Explanation:
(b) Attorney general is appointed by the president. He must be a person who is qualified to be appointed
as a judge of Supreme court . In the performance of his duties he has the right of audience in all courts
in the territory of India.
97. Which among the following pair(s) is/are correctly matched?
Spacecraft Purpose
1. New Horizons : Orbiting the Mercury and transmitting data to earth.
2. MAVEN : study of Mars
3. Messenger : flyby study of the Pluto system
Select the correct answer using the codes given below.
a)
b)
c)
d)

2 only
1 and 2 only
2 and 3 only
None of the above

Correct Answer: A
Your Answer: Unanswered
Explanation:
(a) In this question, purpose of New Horizons and Purpose of Messenger has been inter-changed.
98. Which among the following countries rank above India on Social Progress Index (SPI)?
1. Sri Lanka 2. Bangladesh
3. Nepal 4. Pakistan
Select the correct answer using the codes given below:
a) a ) 1 and 2 only
b) 1 and 3 only
c) 1, 2 and 3 only
d) 1, 2 and 4 only
Correct Answer: C
Your Answer: Unanswered
Explanation:

ForumIAS
Web: http://www.forumias.com/
Email: ravi@forumias.com

Page 76

Score: 0.00 / 200

Exam Title : GS Simulator Test 5


Email

Exam URL : http://compete.forumias.com

Q.98) Explanation 98 : The Social Progress Index measures the extent to which countries provide for the
social and environmental needs of their citizens. Fifty-four indicators in the areas of basic human needs,
foundations of wellbeing, and opportunity to progress show the relative performance of nations. Indias
rank is 101. Sri Lankas rank is 88. Nepals rank is 98. Bangladeshs rank is 100. Pakistans rank is 122.
99. Consider the following diseases.
1. Athletes Foot
2. Chicken Pox
3. Ringworm
Which of the above disease(s) is/are caused by Fungi?
a)
b)
c)
d)

3 only
1 and 3 only
2 and 3 only
1, 2 and 3

Correct Answer: B
Your Answer: Unanswered
Explanation:
(b) Common Diseases Caused by Fungi:
1) Fungal Meningitis
2) Aspergillus Infection
3) Athlete's Foot
4) Jock Itch
5) Ringworm.
Chicken Pox is caused by virus named Varicella.
100. The term Compulsory Licensing is associated with
a) WIPO
b) TRIPS
c) GATS
d) Proposed India EU Free Trade Agreement
Correct Answer: B
Your Answer: Unanswered
Explanation:

ForumIAS
Web: http://www.forumias.com/
Email: ravi@forumias.com

Page 77

Score: 0.00 / 200

Exam Title : GS Simulator Test 5


Email

Exam URL : http://compete.forumias.com

Compulsory licensing is when a government allows someone else to produce the patented product or
process without the consent of the patent owner.
It is one of the flexibilities on patent protection included in the WTOs agreement on intellectual property
the TRIPS (Trade-Related Aspects of Intellectual Property Rights) Agreement.

ForumIAS
Web: http://www.forumias.com/
Email: ravi@forumias.com

Page 78

Score: 0.00 / 200

Вам также может понравиться